BarBri Contracts MBE Questions

Lakukan tugas rumah & ujian kamu dengan baik sekarang menggunakan Quizwiz!

A builder contracted to build a house for a newly married couple. Terms of the contract provided that the builder would receive the contract price when the building was fully completed. Just when the builder had completed one-half of the structure, a tornado struck the area and demolished the building. What is the builder entitled to recover from the couple under the contract? (A) Nothing. (B) One-half of the contract price. (C) One-half of the fair market value of what remains of the house. (D) Cost of materials and reasonable labor costs.

(A) The builder will not be able to recover anything from the couple under the contract because he has not performed his duty. Under the parties' contract, the builder's completion of the house was a condition precedent to the couple's duty to pay. The condition precedent was not discharged by the destruction of the work in progress because construction has not been made impossible, but rather merely more costly—the builder can rebuild. Thus, he is not entitled to any recovery. Note, however, that a number of courts will excuse timely performance because the destruction was not the builder's fault. (B) is incorrect because the contract is not divisible (i.e., it is not divided into an equal number of parts for each side, each part being the quid pro quo of the other); thus, completion of one-half of the house did not entitle the builder to one-half of the price. (C) is incorrect because it is not a correct measure of recovery. As stated above, the builder cannot recover under the contract. However, he could recover restitution if he determined that he could not perform under the contract by rebuilding. Restitution is a remedy that prevents unjust enrichment by imposing on a recipient of requested goods or services a duty to pay for the benefit received when there is a failed contract or no contractual relationship between the parties. The measure of recovery here would be the fair market value of what remains of the house because that is the benefit conferred—it would not be cut in half merely because the house was only half completed. (D) is an incorrect contract recovery because the builder has not fulfilled the condition precedent to the couple's duty to pay. The only way the builder could recover anything would be in an action for restitution. Sometimes, in cases where there is little or no benefit to the other party, the measure of restitutionary recovery is the detriment suffered by the plaintiff. However, when the plaintiff is in breach, the courts that permit recovery limit it to the contract price less damages caused by the breach. Because the builder only did half of the work, he is in breach by not rebuilding the house. Hence, the couple's damages, which involve building an entirely new house, outweigh any possible restitution to the builder.

On June 1, an uncle whose niece was looking to buy a car wrote a letter to a car dealership, stating that he would guarantee payment of the purchase price for any car that it sold to his niece costing less than $10,000. On June 3, after receiving this letter, the dealership sold a $9,500 car to the niece for $2,000 down, with the balance to be paid by the end of the year. The uncle died unexpectedly on June 17. The dealership was unaware of the uncle's death when it mailed the uncle a letter on June 19 accepting his offer. The letter also stated that the niece was a good customer in the past and that it had, in fact, planned to extend her credit for the purchase before receiving the uncle's letter. In August, the niece was killed and her estate is bankrupt. Can the dealership succeed in an action against the uncle's estate for the balance of the price of the car when it becomes due? (A) Yes, because the dealership had accepted the uncle's offer before the uncle died. (B) Yes, because the dealership reasonably, justifiably, and foreseeably relied on the uncle's promise. (C) No, because the dealership would have sold the car to the niece even without the uncle's promise. (D) No, because the uncle died before the dealership mailed the letter notifying him that it had accepted his offer.

(A) The car dealership will succeed. The uncle made an offer, which was accepted by the car dealership by doing the requested act. The uncle's offer was to guarantee the purchase price, up to $10,000, of a car in exchange for the car dealership's selling a car to the niece. This offer traditionally could be accepted by an act (the sale of the car), but could also be accepted by a promise to sell the car to the niece. When a bilateral contract is accepted by performance, the offeree must give the offeror notice within a reasonable time or the offeror may be discharged. Here, the car dealership accepted the offer by selling the car, and a contract was formed at that time. Although an acceptance must be communicated to the offeror, it is not always necessary for the offeree to be the party to communicate it. It would be reasonable to assume that the niece would communicate the fact of the sale to the uncle. The dealership would have no obligation to do more unless it had reason to know that the uncle (the offeror) had no adequate means of learning of the performance with reasonable promptness. Here, the dealership went beyond that by sending notice within a reasonable time. Therefore, when the niece did not pay for the car, the dealership could collect from the uncle or his estate. (B) is wrong because it relies on a promissory estoppel theory, but here the uncle's promise is supported by a bargained-for exchange (his promise in exchange for the car dealership's sale); thus, there is no need to rely on promissory estoppel. (C) is wrong because the car dealership's motive in selling the car to the niece is not relevant. The car dealership did something it was not under a duty to do (sell the car to the niece) and that is sufficient consideration to support the uncle's promise to guarantee payment. (D) is wrong because the offer was accepted when the car dealership did the act, and the uncle was alive at that time.

A contractor with a contract to deepen a well in a drought-stricken area mistakenly entered onto the wrong property and proceeded to deepen the well there. The owner of the property saw the contractor at work but said nothing. When the contractor completed the job, the property owner refused to pay his bill, and the contractor filed suit. In her answer, the property owner stated that she thought the contractor was employed by the county and that the government was paying for the work because of the drought. She knew, however, that two of her neighbors had recently paid private contractors to deepen their wells. Which of the following arguments offers the contractor his best chance for winning his lawsuit? (A) Implied-in-fact contract. (B) Promissory estoppel. (C) Mutual mistake. (D) Unilateral mistake

(A) The contractor's best (and only) argument would be that the property owner's silence while the contractor deepened her well was an acceptance by silence of an implied-in-fact contract. An implied-in-fact contract is formed by manifestations of assent other than oral or written language, i.e., by conduct. Where a person knowingly accepts offered benefits, such conduct, viewed objectively, may be said to manifest an agreement to the conferral of such benefits, resulting in a contract implied in fact. While generally an acceptance must be communicated to an offeror to be effective, courts will often find an acceptance where an offeree silently accepts offered benefits. Here, the property owner's purported belief that the work was being done by a county employee at no charge is not plausible. The property owner had no prior notification from or contact with any county employee, and it is not reasonable to believe that anyone, including an employee of the government, would enter upon and disturb private property without prior consent. Moreover, the property owner knew that her neighbors had not had their wells deepened by county workers but had paid private contractors to do the work. Thus, the facts strongly suggest that the property owner's silence as she watched the contractor deepen her well was acceptance by silence of an implied-in-fact contract. (B) is incorrect because promissory estoppel is inappropriate in this case. Under the doctrine of promissory estoppel, as outlined in section 90 of the First Restatement, a promise is enforceable to the extent necessary to prevent injustice if the promisor would reasonably expect to induce action or forbearance of a definite and substantial character and such action or forbearance is in fact induced. Here, the contractor and the property owner whose well he deepened had never met or negotiated for services and had no communications with one another prior to the contractor sending the property owner his bill, and the contractor did not even realize that he was doing the work for that property owner's benefit. Thus, promissory estoppel is not proper. (C) and (D) are incorrect because mistake is a defense to formation of a contract and is raised to render a contract voidable by the adversely affected party. Thus, mistake would not be a ground on which relief could be granted. Even if the property owner's purported mistake were treated as credible by the court, there was no "mutual" mistake because the contractor and the property owner were not mistaken about the same fact, but rather, each was mistaken about a different fact (the contractor, that he was at the right address, and the property owner, that the contractor was a county employee doing the work without charge). Nor could the facts be characterized as a unilateral mistake, in which only one of the parties is mistaken about facts relating to the agreement, because both parties were operating under a mistaken belief. Thus, both (C) and (D) are incorrect.

A wholesale seller of widgets telephoned a retail seller of widgets and told him that he had 5,000 pounds of widgets ready for delivery at $5,000. The retailer agreed to purchase the widgets, but stated that he wanted the wholesaler to deliver 2,000 pounds now and 3,000 pounds next month. There were no further communications between the parties. Assuming that the retailer's request is not a material change of terms, what is the most likely result of the conversation between the wholesaler and the retailer? (A) A contract was formed to deliver 2,000 pounds now and 3,000 pounds next month. (B) A contract was formed to deliver 5,000 pounds now. (C) No contract was formed, because the retailer's response was merely a counteroffer and a rejection. (D) No contract was formed, unless the wholesaler notified the retailer within a reasonable time of his assent to the proposed schedule of delivery

(A) The conversation created a contract for 2,000 pounds of widgets now and 3,000 pounds next month. Because the contract is for the sale of goods, the UCC governs. Under the UCC, a contract is formed whenever it appears from the parties' communications that they intended to enter into a contract. Here, it is clear that the parties intended to enter into a contract, but the acceptance contained terms additional to the offer terms. When this occurs, the UCC provides for which terms govern: If the contract is between merchants, the additional terms in the acceptance are included in the contract, unless (i) the additional terms materially alter the contract, (ii) the offer expressly limits acceptance to the terms of the offer, or (iii) the offeror objects within a reasonable time. Here, both parties are merchants, and we were told to assume that the delivery terms do not materially alter the contract. There is no indication that the offer limited acceptance to the terms of the offer or that the wholesaler objected to the terms; thus, there is a contract containing the additional terms. (B) would be correct if one of the parties were not a merchant, because under the UCC, when an acceptance proposes additional terms, a contract would be formed under the terms of the offer unless both parties are merchants. (C) would be correct if the UCC did not apply, because under the common law, an acceptance must mirror the offer (the "mirror image" rule); if new terms are added in the acceptance, it is treated as a counteroffer. (D) is incorrect because, under the UCC, no notice was necessary to form the contract. Notice would be required, however, if the wholesaler did not want to be bound by the additional terms. Note that the fact that this was an oral conversation does not prevent formation of the contract. To be enforceable, any contract for the sale of goods priced at $500 or more must be evidenced by a writing signed by the party to be charged. This affects enforceablility, not formation. For this contract to be enforceable, some form of signed writing (e.g., a merchant's confirmatory memo) would be necessary.

An advertising agency specializing in aerial banners and skywriting signed a contract with a film production company that was premiering a new blockbuster film. The contract provided that the agency would advertise the film by flying over the city towing a giant streamer belonging to the film company heralding the film's catch phrase and title in large letters. This contract specified that the flight was to be conducted on the first Saturday in June at noon (the day of the local premier), and the film company was to pay the advertising agency $500 for the flight. On the designated Saturday, the advertising agency was unable to fly because of a defective fuel pump. The defective condition was entirely unforeseeable and did not occur through any negligence or fault of the agency. The film company did not pay the agency, and each of the parties has sued the other for damages. Which of the following best states the rights and liabilities of the parties? (A) The film company is entitled to recover damages from the advertising agency on account of the agency's failure to fly. (B) The advertising agency is entitled to recover from the film company the $500 contract price, as the incapacity of the airplane was not the agency's fault. (C) Neither party is entitled to recover against the other, because the advertising agency's duty to fly was discharged by impossibility, and the film company's duty to pay was contingent on the agency's flight. (D) Neither party is entitled to recover against the other, because the film company's offer to pay $500 for the flight was in effect an offer for an act, and because the act was not performed, there was no valid acceptance.

(A) The film company will be able to recover damages from the advertising agency because the agency's failure to fly constituted a breach of contract. The parties entered into a bilateral contract—the agency promised to fly with the streamer and the film company promised to pay for the flight. The agency breached the contract by failing to fly on the designated Saturday. Its duty to fly was not discharged by impossibility. A contractual duty to perform may be discharged by 10. Contracts Answers objective impossibility (i.e., no one could have performed), but subjective impossibility (defendant could not perform) is insufficient. Here, the defect in the plane constituted only subjective impossibility (if it amounted to impossibility at all) because the agency could have obtained another plane to pull the streamer. If the agency had been unable to fly the plane because of weather (e.g., a severe ice storm), its performance would have been objectively impossible, and the agency would have been discharged. However, under these facts, the film company is entitled to damages for the agency's breach. (B) is incorrect because the film company's duty to perform (pay $500) was subject to the condition precedent of the agency's performance (flying), and, as discussed above, the agency breached the contract by failing to fly. Therefore, the film company's duty to pay never arose. The fact that the engine problem was not the agency's fault does not change things. The agency's inability to perform, even if it were due to impossibility, would merely discharge the contract, and each party would be excused from performance; the film company would not have to pay the $500. (C) is incorrect because, as determined above, the agency's duty was not discharged because performance was still possible. (If there had been objective impossibility, (C) would have been the correct choice.) (D) is incorrect because it suggests that the contract was a unilateral one (the offer to pay could be accepted only by completion of performance). This interpretation is clearly contrary to the facts. Although the film company offered to pay $500 for the flight, the agency accepted that offer by signing the contract. A promise to pay was given in exchange for a promise to fly. Thus, there was a contract to which both parties were bound.

In March, a homeowner contracted with a buyer to sell his house for $280,000, with the purchase price to be paid and the deed to be delivered on July 1. On May 1, the buyer wrote the homeowner a letter, stating that she decided to buy another house instead. Which of the following is true? (A) The homeowner may sue the buyer upon receipt of the letter. (B) The homeowner must wait until July 1 to sue the buyer, the date on which the purchase price is to be paid. (C) The homeowner must make a written demand to the buyer seeking adequate assurance of performance and wait a reasonable time for a response prior to filing suit. (D) The homeowner cannot sue the buyer because the parties' promises are executory at this stage.

(A) The homeowner may sue the buyer when he receives the letter because an anticipatory breach situation exists. Anticipatory repudiation occurs where a promisor, prior to the time set for performance of her promise, indicates that she will not perform when the time comes. Anticipatory repudiation serves to excuse conditions if: (i) there is an executory bilateral contract with executory duties on both sides; and (ii) the words or conduct of the promisor unequivocally indicates that she cannot or will not perform when the time comes. The requirements for anticipatory repudiation are met here because the homeowner's duty to deliver the deed and the buyer's duty to pay have yet to be performed, and the buyer's writing unequivocally states that she will not pay unless the homeowner performs extra tasks. In the case of anticipatory repudiation, the nonrepudiating party has the option to treat the contract as being breached and sue immediately. Therefore, the homeowner may sue the buyer upon receipt of the letter. (B) is incorrect because, as stated above, the homeowner need not wait until July 1 to sue because the buyer's letter amounts to anticipatory repudiation. The nonrepudiating party may treat this as a total repudiation and 12. Contracts Answers sue immediately. The doctrine of anticipatory repudiation would not apply if both sides did not have executory duties to perform. In such a case, the nonrepudiator must wait to sue until the time originally set for performance by the repudiating party. However, as discussed above, the homeowner's duty was still executory at the moment of the buyer's repudiation. Therefore, the homeowner does not have to wait until July 1 to sue the buyer for breach. (C) is incorrect because seeking adequate assurances of the buyer's intent to perform is not necessary when the repudiating party has stated unequivocally that she will not perform. A party may in writing demand adequate assurance of due performance if "reasonable grounds for insecurity arise with respect to the performance of" the other party, after which the nonrepudiating party can treat the contract as repudiated if those assurances are not given within a reasonable time. However, here, it is clear from the buyer's writing that she is unwilling to perform and, therefore, the homeowner need not seek assurances of an intent to perform. (D) is incorrect because the homeowner can sue the buyer immediately because their promises are still executory. As discussed above, the doctrine of anticipatory repudiation is applicable only if there are executory duties on both sides. Here, the homeowner's duty to deliver a deed and the buyer's duty to pay are both executory, so the doctrine applies and the homeowner can sue the buyer now.

A homeowner and a builder entered into a written contract to build a sauna in a spare room in the homeowner's home at a cost of $3,000. The contract contained a clause stating that the builder will not begin construction without prior approval of the plans by the homeowner's certified public accountant. The builder submitted his designs to both the homeowner and the accountant. The homeowner liked the plans, but the accountant did not and withheld his approval. The builder asked the homeowner whether she wanted him to submit new designs. The homeowner told the builder orally, "No! Your designs are great! My accountant is crazy! You go right ahead and construct the sauna." The builder constructed the sauna. The homeowner now refuses to pay the builder, citing the clause requiring approval by the accountant. If the builder sues the homeowner, what will the builder likely recover? (A) The full contract price, because the accountant's approval was not a condition precedent for the contract to take effect. (B) The full contract price, because once the builder began building the sauna after speaking to the homeowner, the homeowner did nothing to stop the builder. (C) The reasonable value of the builder's services and materials, because otherwise the homeowner would be unjustly enriched. (D) Nothing, because the homeowner's oral statement will be excluded by the parol evidence rule.

(B) By her statement to the builder, the homeowner waived the benefit of the condition requiring the accountant's approval of the design plans, and the builder detrimentally relied on the statement by building the sauna. Thus, there is a binding waiver of the condition. A condition is an event, other than the passage of time, the occurrence or nonoccurrence of which creates, limits, or extinguishes the absolute duty to perform in the other contracting party. The occurrence of a condition may be excused under a number of different circumstances. One such circumstance is where the party having the benefit of the condition indicates by words or conduct that she will not insist upon it. If a party indicates that she is waiving a condition before it happens, and the person affected detrimentally relies on it, a court will hold this to be a binding estoppel waiver. The promise to waive the condition may be retracted at any time before the other party has detrimentally changed his position. Here, the contract provided that the builder could not begin work without the accountant's prior approval. This approval was a condition that had to be met before the homeowner's duty to pay would arise. When the homeowner told the builder to commence working on the sauna, even though the accountant had withheld his approval, the homeowner was telling the builder that she was waiving the condition of the accountant's approval. The builder then acted in detrimental reliance on this statement by in fact starting and completing the building of the sauna. While the homeowner could have retracted her statement and reinstated the condition prior to the builder's detrimental reliance, she did nothing when the builder began working on the sauna. Under such circumstances, the homeowner made a binding waiver of the condition and will be estopped from asserting it. Thus, the builder is entitled to recover the full contract price. (A) is incorrect because, as discussed above, the accountant's approval was a condition precedent for the parties' contractual duties to arise. The builder's duty to build the sauna and the homeowner's duty to pay for it would not arise without the condition of the accountant's approval either being satisfied or being excused. (C) is incorrect because unjust enrichment is a quasi-contract alternative that the builder could utilize if he did not have a contract remedy. Here, however, the builder can recover the full contract price because the homeowner waived the condition and is estopped from retracting the waiver. (D) is incorrect because the parol evidence rule does not Contracts Answers 11. prohibit evidence of a subsequent modification of a written contract; the rule applies only to prior or contemporaneous expressions. Consequently, it may be shown that the parties altered the integrated writing after its making. The oral agreement between the homeowner and the builder described in the facts was made subsequent to the writing. Therefore, the parol evidence rule is inapplicable to this agreement.

A recent nursing school graduate mailed a letter to a classmate on July 1 telling her that she was moving to take a nursing position in another city and asking her whether she wanted "the stuff in my house" for $2,500. The classmate received the letter on July 2, and on July 3 she sent the newly minted nurse a letter accepting the offer. The next day the classmate changed her mind, called the nurse, and told her to forget the deal. Later that same day, the nurse received the letter that her classmate had sent on July 3. Is there a contract between the nurse and her classmate? (A) Yes, because the contract is for the sale of goods for more than $500 and the classmate's attempted rejection is oral. (B) Yes, because the classmate's letter of acceptance was effective when she mailed it. (C) No, because the classmate's rejection was communicated to the nurse before her letter of acceptance was received. (D) No, because the description of the subject matter as "the stuff in my house" is not sufficiently definite and certain.

(B) The classmate accepted the nurse's offer when she mailed the letter on July 3; thus, a contract was formed. Under the mailbox rule, acceptance of an offer by mail creates a contract at the moment the acceptance is posted, properly stamped, and addressed. If the offeree sends both an acceptance and a rejection, whether the mailbox rule will apply depends on which the offeree sent first, the acceptance or the rejection. If the offeree first sends an acceptance and later sends her rejection, the mailbox rule does apply. Thus, even if the rejection arrives first, the acceptance is effective upon mailing (and so a contract is formed) unless the offeror changes his position in reliance on the rejection. Here, the classmate first sent an acceptance, then called with her rejection. The mailbox rule applies, and because there is nothing in the facts to show that the nurse relied on the rejection, a contract was formed. (A) is wrong because it implies that a rejection must be in writing. There is no such requirement. Also, the rejection (absent detrimental reliance) has no effect on the contract because the offer had already been accepted and the contract formed. (C) is wrong because, as stated above, under the mailbox rule the fact that the rejection was received before the acceptance is irrelevant (unless there has been detrimental reliance on the rejection, which was not the case here). The contract was formed when the classmate sent her acceptance. (D) is wrong because the description, although somewhat ambiguous, can be made reasonably certain by evidence of the subjective understanding of the parties and extrinsic evidence of what was in the house, which a court will consider to clarify an ambiguous term.

On November 5, an electronics store owner realized that his stock of 15 copies of the most popular video game of the holiday shopping season would not last until the first of the next month. Seeing an advertisement from the manufacturer of the game in a trade journal listing its price at $3,000 per hundred, with delivery one week from order, the store owner e-mailed to the manufacturer an order for 100 copies of the game at $3,000 per hundred. There were no further communications between the store owner and the manufacturer. By November 25, the store owner realized that the manufacturer was not going to deliver any of the video games. He thus was forced to obtain additional stock by purchasing from a middleman at a cost of $4,000 per hundred. The store owner brings an action for breach of contract against the manufacturer. Who will prevail? (A) The manufacturer, because the communications between the parties were not definite or certain enough to form a contract. (B) The manufacturer, because it never accepted the offer contained in the store owner's e-mail. (C) The store owner, because his e-mail was an acceptance of the manufacturer's offer. (D) The store owner, because he changed his position in reliance on the manufacturer's promise to deliver the video games within one week.

(B) The manufacturer will prevail because it never accepted the offer. For a communication to be an offer, it must contain a promise, undertaking, or commitment to enter into a contract, rather than a mere invitation to begin negotiations. The broader the communicating media, e.g., publications, the more likely it is that the courts will view the communication as merely the solicitation of an offer. An advertisement in a trade journal generally is construed as an invitation to submit offers, not an offer itself. It is an announcement of the price at which the seller is willing to receive offers. Thus, the store owner's e-mail was an offer that was never accepted by the manufacturer. (A) is wrong because the communications were definite and certain enough to form a contract. Under the UCC, only the quantity term must be definite and certain (or capable of being made so). Here, the quantity term, 100, was clear; the problem is that the offer containing it was not accepted. (C) is wrong because, as discussed above, the ad is not an offer. (D) is wrong because 2. Contracts Answers the ad in the trade journal was not a promise; hence, the store owner cannot rely on promissory estoppel or detrimental reliance to recover.

A landlord heard that a company was looking for old buildings to purchase and raze to make room for new development. On July 1, the landlord sent a letter to the company stating, "I own two adjacent apartment buildings that might meet your needs. One is located on 123 Main Street and the other property is right behind it on 123 Wood Street. I will sell you one or both of the apartment buildings for $250,000 each. This offer remains open until August 1." On July 15, the company faxed the landlord, "Accept your offer with respect to the apartment building on Main Street." On July 18, the company faxed the landlord, "Will also take the building on Wood Street." On July 22, the landlord discovered that he did not have good title to the Wood Street property. Which of the following would provide the best legal support to the landlord's contention that he was not liable for breach of contract as to the Wood Street property? (A) Impossibility of performance. (B) Unilateral mistake as to basic assumption. (C) Termination of the offer by the company's having first contracted to buy the Main Street property. (D) Excuse by failure of an implied condition precedent.

(C) Arguably, the company's fax on July 15 could reasonably be interpreted as a rejection of the landlord's offer as to the Wood Street building, i.e., having accepted only the Main Street property, the company was impliedly rejecting the rest. Once rejected, the offer is terminated and the offeree's power of acceptance is extinguished; thus, the July 18 attempt to accept would be ineffective. None of the other alternatives makes sense. For impossibility of performance in (A) to apply, the impossibility must be "objective"; i.e., the duties could not be performed by anyone. Also, the impossibility must arise after the contract has been entered into. (D) is wrong because a condition precedent must be distinguished from a promise. A condition is the occurrence of an event that will create, limit, or extinguish the absolute duty to perform. In this case, it would probably be determined that the intention of the parties was an exchange of promises. (B) is wrong because a unilateral mistake in most cases will not prevent formation of a contract. Only mutual mistake going to the heart of the bargain may prevent the formation of the contract.

A steel mill contracted with an appliance manufacturer to sell the manufacturer a 10-ton coil of steel. The written contract specified the quality of the steel to be delivered and also contained a clause that made the agreement contingent upon the appliance manufacturer obtaining a letter of credit from a federally insured bank at an interest rate of no more than 2.5%. The appliance manufacturer subsequently received a letter of credit from a federally insured bank at 3% interest. Upon hearing of this, the mill refused to ship the steel to the appliance manufacturer. If the appliance manufacturer sues the mill for breach, is the appliance manufacturer likely to prevail? (A) No, because the condition was not satisfied. (B) No, because the appliance manufacturer breached its duty of good faith. (C) Yes, because the interest-rate term was included in the contract for the appliance manufacturer's benefit. (D) Yes, because the .5% differential between the interest rate stated in the contract and the rate actually obtained is not material.

(C) The appliance manufacturer will prevail because the limitation on the interest rate was placed in the contract for his protection and he was free to waive it. The terms of the contract made the contract contingent upon the appliance manufacturer obtaining a letter of credit from a federally insured bank. Thus, the letter of credit was a condition precedent to the contract. The limit on interest was clearly for the appliance manufacturer—he would not have to enter into the contract unless he could find a satisfactory loan. One having the benefit of a condition may by words or conduct indicate that he will not insist upon the condition, and the courts will enforce such a waiver. (A) is incorrect because while the 2.5% condition was not met, it was placed in the contract for the appliance manufacturer's benefit, and he was free to waive it. (B) is incorrect because, while the UCC (which governs here because we have a contract for the sale of goods) places a duty of good faith on all parties, it is not a breach of the duty for a party to waive a condition that was placed in the contract for that party's benefit. The appliance manufacturer sought and obtained a loan; so it acted in good faith. (D) is incorrect because while the extra .5% may be immaterial, it would be sufficient as a condition to prevent the contract if the term were not for the manufacturer's benefit. The court would give the condition a literal meaning, and the fact that the variance was small would not alter the fact that the condition precedent was not met.

The owner of a summer house entered into a written agreement with a plumber. The contract contained a clause requiring all plumbing work to be completed by noon on June 1 and provided that the homeowner would pay the plumber $1,200 for his work. The plumber began working on the job on May 28. When he quit working for the day on the afternoon of May 29, half of the job was completed. Shortly thereafter, a heavy rain began, which caused a flash flood. The house was swept away in the flood waters. Which of the following best describes the obligations of the plumber and the homeowner after the flood? (A) Neither the plumber nor the homeowner is discharged from their obligations under the contract. (B) The homeowner is obliged to pay the plumber $1,200. (C) The plumber is discharged from his obligation but is entitled to recover from the homeowner the fair value of the work he performed prior to the flood. (D) Neither the plumber nor the homeowner has any further obligations.

(C) The destruction of the house discharges the plumber's duties due to impossibility, but the plumber has a right to recover for the reasonable value of the work he performed. Contractual duties are discharged where it has become impossible to perform them. The occurrence of an unanticipated or extraordinary event may make contractual duties impossible to perform. If the nonoccurrence of the event was a basic assumption of the parties in making the contract, and neither party has assumed the risk of the event's occurrence, duties under the contract may be discharged. If there is impossibility, each party is excused from duties that are yet to be performed. If either party has partially performed prior to the existence of facts resulting in impossibility, that party has a Contracts Answers 9. right to recover in quasi-contract for the reasonable value of his performance. While that value is usually based on the benefit received by the defendant (unjust enrichment), it also may be measured by the detriment suffered by the plaintiff (the reasonable value of the work performed). Here, the house on which the plumber was to perform plumbing repairs was totally destroyed in a flood. The facts indicate that this flood was of such an unexpected nature that its nonoccurrence was a basic assumption of the parties, and neither party was likely to have assumed the risk of its occurrence. Thus, it has become objectively impossible for the plumber (or anyone else) to complete the job. This impossibility will discharge both the homeowner and the plumber from performing any contractual duties still to be fulfilled. Therefore, the plumber need not finish the repair work, and the homeowner is not obligated to pay the entire amount of $1,200. However, the plumber can recover under quasi-contract. (A) is incorrect because both parties are discharged. (B) is incorrect because, as discussed above, the homeowner is obligated to pay for the value of the plumber's services to date, not the full contract price. (D) is incorrect because it fails to account for the fact that the homeowner will have to pay the plumber for the value of the work already performed.

A general contractor who wished to bid on a construction project solicited bids from a variety of subcontractors. Four electrical subcontractors submitted bids to the contractor in the amounts of $75,000, $85,000, $90,000, and $95,000, respectively. As he was making out his company's bid, which was higher than he wanted it to be, the contractor called the low bidder on the electrical work and told him, "We won't be able to do it with your present bid, but if you can shave off $5,000, I'm sure that the numbers will be there for us to get that project." The low bidder told the contractor that he could not lower his bid, adding that the bid he submitted was based on a $15,000 error, and he could not do the job for less than $90,000. The contractor lost the construction job and subsequently sued the low bidder. For what is the low bidder is liable? (A) Breach of contract, because the mistake was not so unreasonably obvious as to make acceptance of his bid unconscionable. (B) Breach of contract, because the mistake was unilateral. (C) Nothing, because the low bidder rejected the contractor's counteroffer. (D) Nothing, because even though the low bidder lacked authority to renege on its bid, the contractor suffered no damages because no bidder was willing to do the work for $70,000.

(C) The low bidder is liable for nothing because no contract was formed between the contractor and the low bidder. Formation of a contract requires mutual agreement between the parties (offer and acceptance) and consideration. There was no contract here because there was no acceptance. The low bidder's bid constituted an offer—a certain and definite promise, undertaking, or commitment to enter into a contract communicated to the offeree. An offer gives the offeree the power to accept and create a contract until the offer is terminated. An offer can be terminated in a number of ways, including through a counteroffer from the offeree. A counteroffer serves as both a rejection terminating the original offer and a new offer from the original offeree, thus reversing the former roles of the parties and giving the original offeror the right to accept or reject the new offer. Here, the contractor's call constituted a rejection and a counteroffer that the low bidder rejected, and so no contract was formed. Therefore, the low bidder cannot be held liable. (A) and (B) are incorrect because, as stated above, a contract was never formed between the contractor and the low bidder. Thus, it is irrelevant whether the mistake was unilateral or obvious. (Note that the general rule is that a contract will not be set aside for a unilateral mistake unless the nonmistaken party either knew or should have known of the mistake. Thus if the contractor had not called the low bidder but had instead accepted his bid, the low bidder would be liable on the contract despite his mistake because (B) the mistake was unilateral and (A) it was not obvious.) (D) is incorrect for several reasons: First, it relies on the existence of a contract, and as stated above, there is no contract here. Second, the premise that the low bidder could not renege on its offer is untrue. The general rule is that offers are revocable until accepted. In a subcontractor bid situation, a bid is treated as irrevocable for a reasonable amount of time because of detrimental reliance (i.e., the general contractor will rely on the mistaken bid in preparing his bid). However, here the contractor learned of the low bidder's mistake before any reliance on the bid. Moreover, it is unclear whether the contractor is complaining about the low bidder's reneging on his bid; the contractor appears to be complaining that the low bidder refused to lower the mistaken bid. The low bidder would have no duty to lower its bid in any case. The final premise in (D) is irrelevant. If the low bidder lacked the power to renege, the lack of power goes to the $75,000 bid; the fact that no one would do the job for $70,000 has no bearing on the issue.

The owner of a sporting goods store noticed that her tent stock was running low. After consulting various manufacturers' catalogs, she decided to order from a large manufacturer of camping equipment whose catalog listed the 9 x 12 tent that she wanted at a cost of $70 per tent. On April 1, the store owner phoned the manufacturer and placed her order for 10 tents with the manufacturer's sales agent. The next day, the manufacturer mailed the store owner a signed letter stating that there was a contract for sale for 10 tents at $70 per tent plus a $40 shipping fee, and that the tents would be shipped on April 16. The store owner received the letter on April 4, but she never responded. On April 15, she received a catalog from another tent company showing tents similar to the ones that she ordered, but for a cost of only $60. The store owner immediately called the manufacturer with whom she had placed her order to cancel it. Nevertheless, the manufacturer shipped the tents to her on April 16. If the manufacturer sues the store owner to enforce the contract, is the manufacturer likely to prevail? (A) No, because there was no meeting of the minds regarding the price term. (B) No, because the store owner's promise was not in writing. (C) Yes, because the manufacturer's April 2 letter was sufficient to bind the store owner. (D) Yes, because the price of each tent is less than $500.

(C) The manufacturer will prevail because the April 2 letter was sufficient to bind the store owner. There are two issues involved here, the battle of the forms provisions of the UCC and the Statute of Frauds. Generally, to be enforceable, a contract for the sale of goods priced at $500 or more must be evidenced by a writing signed by the party sought to be bound. However, in contracts between merchants, if one party, within a reasonable time after an oral agreement has been made, sends to the other party a written confirmation of the understanding that is sufficient under the Statute of Frauds to bind the sender, it will also bind the recipient if: (i) the recipient has reason to know of the confirmation's contents; and (ii) she does not object to it in writing within 10 days of receipt. Here, both parties are merchants: one is a store owner and the other a manufacturer. The day after the store owner's oral order was placed, the manufacturer mailed a confirmation letter. Assuming the letter was signed or on the manufacturer's letterhead, it would be sufficient to bind the sender. Since the store owner received the letter and read it, she had reason to know of its contents and did not respond. Thus, the writing is sufficient under the Statute of Frauds to bind the store owner as well as the manufacturer. There is also an issue with the fact that the confirmatory memo contains a new term regarding shipping fees. If both parties to the contract are merchants, additional terms in an acceptance or confirmation will be included in the contract unless: (i) they materially alter the original terms of the offer (e.g., they change a party's risk or the remedies available); (ii) the offer expressly limits acceptance to the terms of the offer; or (iii) the offeror has already objected to the particular terms, or objects within a reasonable time after notice of them is received. The shipping term does not materially alter the terms of the offer, the offer did not limit the acceptance to its terms, and the store owner has not objected. Thus, the new term will become part of the contract. (A) is wrong because the shipping term is separate from the price term. There was a meeting of the minds as to price. (B) is wrong because the confirmatory memo binds the store owner despite the fact that she did not sign a writing. (D) is wrong. The Statute of Frauds applies here because the total price of the contract was for $500 or more. However, as discussed above, the April 2 letter was sufficient to bind the store owner.

A doll collector knew that an acquaintance from her doll collectors' club coveted one particular doll that she owned. The doll collector mailed a letter to the acquaintance on May 3 offering to sell the doll to her for $750. Her letter arrived on May 4. On May 5, the doll collector changed her mind and immediately mailed a revocation to the acquaintance. This revocation arrived on May 7. As the mail carrier handed it to her, the acquaintance simultaneously handed to the mail carrier her own letter to the doll collector, unequivocally accepting her offer. What is the result of the actions here? (A) The revocation was effective upon mailing, and the acceptance would be treated as a counteroffer. (B) The acceptance was effective, as long as the acquaintance had no knowledge of the contents of the doll collector's letter when she handed her letter to the mail carrier. (C) The outcome would turn on the court's determination as to whether the doll collector's letter had been received by the acquaintance before she had entrusted the letter of acceptance to the mail carrier. (D) Handing a letter to a mail carrier is not a proper posting of the acceptance, and hence the acquaintance's purported acceptance is not timely.

(C) The outcome would turn on the court's determination as to whether the doll collector's letter had been received by the acquaintance before she had entrusted the letter of acceptance to the mail carrier. At common law, an acceptance is effective upon dispatch (e.g., upon mailing a properly addressed and stamped letter) under the mailbox rule. The mailbox rule does not apply to revocations, however—revocations are effective only upon receipt. Receipt does not require knowledge of the revocation, but merely possession of it. The communication need not be read by the recipient to be effective. [See Restatement (Second) of Contracts §68] The facts here present a close question as to whether there has been a dispatch of the acceptance before the receipt of the revocation. The outcome of this question will depend on the court's determination as to what came first (the posting of the acceptance or receipt of the revocation). This will decide the existence or nonexistence of the contract. (A) is incorrect because, as indicated above, revocation is effective only upon receipt, not mailing. (B) is incorrect because whether the acceptance is effective depends on whether the revocation was received before the acceptance was dispatched, and whether the revocation was received first is not dependent on whether the acquaintance had knowledge of its contents, but rather it depends on whether she had possession of it. (D) is incorrect because the mailbox rule makes an acceptance effective upon posting, and there is no reason to hold that handing a properly addressed, stamped letter to a mail carrier is not a valid posting.

A general contractor advertised in a trade publication that she planned to bid on the construction of a new building to be located in the town square. The advertisement welcomed bids from subcontractors to perform various functions, including plumbing. The lowest plumbing bid was from a local plumber who bid $10,000. The contractor used the bid in preparing her general bid. One hour after she submitted her general bid, the plumber called her and told her that he made a mistake on the bid he submitted to her, and that he could not do the plumbing work for less than $12,000. The contractor acknowledged that he had done good work for her in the past and said, "I'll just forget you ever made that $10,000 bid." The contractor then hired the second lowest bidder to do the plumbing work for $12,000. If the contractor sues the plumber for damages, will she prevail? (A) Yes, because there was no additional consideration to support a release. (B) Yes, because the dollar amount of the agreement is large enough that the Statute of Frauds applies. (C) No, because a rescission has taken place. (D) No, because the contractor and the plumber mutually agreed to a release.

(C) The unilateral option contract between the plumber and the contractor to keep the plumber's offer open was effectively rescinded by the contractor's expressed intent to make a gift of the obligation owed her. The typical case of rescission involves a bilateral contract where neither party has 4. Contracts Answers yet performed; i.e., the duties of both parties are still executory. However, no contract to do the plumbing work has been created yet, because the contractor has not communicated her acceptance of the bid to the plumber. Despite her use of the plumber's bid to prepare her own bid, the contractor is free to award the plumbing work to someone else if she is awarded the general contract. Hence, the contract to do the plumbing cannot be rescinded because it has not been created. Another contract is present under this fact pattern, however. Under section 87 of the Restatement (Second) of Contracts, the plumber's offer is binding as an option contract because the contractor reasonably relied on it to submit her bid. The option contract here is unilateral: The contractor's acceptance of the option contract by using the bid also constituted performance of her duties under the option contract. In a unilateral contract case, a rescission promise must be supported by either (i) an offer of new consideration, (ii) elements of promissory estoppel (i.e., detrimental reliance), or (iii) the offeree's manifestation of an intent to make a gift of the obligation owed her. The first two alternatives are absent in these facts, but the gift alternative is indicated by the contractor's statement that she will "forget" that the plumber ever made the bid. The contractor's response was an effective rescission of the option contract. (A) is an incorrect choice even though it is a true statement. A discharge of contractual duties by means of a release requires additional consideration or some substitute, such as a signed writing or reliance by the offeror on the discharge. Here there is no additional consideration to support a release, as choice (A) indicates, but the contractor will not win because a rescission has taken place. (B) is incorrect because a large dollar amount for purposes of the Statute of Frauds is irrelevant unless there is a contract for the sale of goods, which must be in writing if the goods are priced at $500 or more. The agreement between the contractor and the plumber involved a contract for services, which is not within the $500 provision of the Statute. (D) is incorrect because, as discussed above, a release requires additional consideration, a signed writing, or detrimental reliance by the offeror. Because none of these is indicated by the facts, a release has not taken place.

A wholesaler persuaded a retailer to order a line of dolls for the Christmas season, even though the retailer was skeptical of the dolls' marketability. The contract provided that the retailer would pay $1,500 for its order of 100 dolls if they sold during the Christmas season. Some dolls did sell, but on February 12, the retailer had 80 of them in inventory. He sent the wholesaler notice that he would be returning the 80 dolls. The wholesaler replied that it did not want the dolls back, that the retailer should continue to try to sell them. Despite this reply, the retailer sent the wholesaler a check for $300 and shipped the dolls to the wholesaler, who refused to accept them but did accept the check. Thereafter, the retailer held the dolls at his warehouse. The wholesaler brought an action to recover the $1,200 balance. Will the wholesaler likely recover? (A) Yes, because the retailer still has the dolls in his possession. (B) Yes, because it was not a condition precedent that the dolls be sold during the Christmas season, but merely a convenient time for payment. (C) No, because sale during the Christmas season was a condition precedent to payment. (D) No, because accepting the $300 constituted a waiver of any rights that the wholesaler may have had to enforce the contract.

(C) The wholesaler will not recover the $1,200 because sale during the Christmas season was a condition precedent. A condition precedent is one that must occur before an absolute duty of immediate performance arises in the other party. Based on the facts here, the intent of the parties was that the retailer would have to pay for the dolls only if they sold during the Christmas season. Sale during that time was a condition precedent to payment. Thus, the retailer had no obligation to pay for the 80 dolls that had not sold by February 12 (well after the Christmas season). Thus, (C) is correct, and (B) is wrong. (A) is wrong because the wholesaler refused the retailer's tender of the dolls, and the retailer is just holding them awaiting the wholesaler's instructions. (D) is wrong because accepting the check did not result in a waiver of any rights the wholesaler may have had. If a monetary claim is uncertain or subject to a bona fide dispute, an accord and satisfaction can be accomplished by a good faith tender and acceptance of a check when that check (or an accompanying document) conspicuously states that the check is tendered in full satisfaction of the debt. Here, there is no indication that the retailer stated that the check was payment in full.

A contractor agreed to build a plant for a manufacturer for $5 million, with $1 million paid in advance and the balance to be paid upon completion of the project. The contract required the contractor to use lighting fixtures from a specific company. Inadvertently, the contractor installed fixtures from a different company. The installed fixtures are generally considered to be of a slightly better quality than the fixtures specified in the contract. The mistake was not discovered until the manufacturer did a final inspection of the building. As built, the plant is worth $10,000 more than it would have been worth had the specified fixtures been used. It would cost the contractor $100,000 to replace the fixtures with the ones specified in the contract. Because of a downturn in the economy, the manufacturer no longer wants to move into the new plant and refuses to pay the contractor because of the breach regarding the light fixtures. If the contractor sues the manufacturer for breach of contract, which of the following doctrines will be most important to a court's decision? (A) Perfect tender. (B) Divisibility. (C) Substantial performance. (D) Quasi-contract.

(C) Under the doctrine of substantial performance, contracts governed by the common law are enforceable despite minor breaches. The contract here—to build a manufacturing plant—is governed by the common law. In determining whether a breach is minor or substantial, courts look to whether the party received the substantial benefit of the bargain. Here, the manufacturer got a plant that was, perhaps, better than the one called for in the contract. Therefore, the contract will be enforceable under the doctrine of substantial performance. (A) is incorrect because the perfect tender doctrine applies only to contracts for the sale of goods—not to common law contracts. (B) is incorrect. Under the divisibility doctrine, if a party performs one of the units of a divisible contract, he is entitled to the agreed-on equivalent for that unit even if he fails to perform the other units. It is not a condition precedent to the other party's liability that the whole contract be performed. For a contract to be divisible: (i) the performance of each party must be divided into two or more parts under the contract, (ii) the number of parts due from each party must be the same, and (iii) the performance of each part by one party is agreed on as the equivalent of the other party's corresponding part. Here, the payment is broken into two parts, but the payments do not correspond to units of work performed by the contractor, so this contract is not divisible. In any case, divisibility is unlikely to be of much help in deciding this case unless the installation of the lights was a separate task under the contract, with a corresponding payment. (D) is incorrect because quasi-contract provides a remedy in some situations in which a contract fails (or is absent) and the defendant would be unjustly enriched as a result. The doctrine is not applicable here because, as discussed above, the contract will not fail due to a minor breach.

A woman lost her dog. She posted signs on many trees and poles throughout her neighborhood for a $500 reward to whomever would find and return her pet. Months later, believing she would never find the dog, the woman adopted a new dog and published an ad in the local newspaper stating that she repealed her previous reward offer. The ad was printed once daily for a week. After the ad had run, a dogcatcher had found the woman's dog, and, because it had no tags, he sent it to the local pound. A volunteer at the pound recognized the woman's dog from one of her reward postings and suggested that the dogcatcher claim the woman's $500 reward. The dogcatcher took the dog to the woman, who refused to pay the reward. If the dogcatcher sues the woman to recover the $500 reward, which of the following, if true, would be the LEAST helpful to the woman's defense? (A) The consideration furnished by the dogcatcher, if any, for the woman's reward promise was legally insufficient under the preexisting duty rule. (B) The dogcatcher was already compensated by the pound for his capturing services. (C) The woman's offer had effectively been revoked prior to the dogcatcher's attempted acceptance. (D) The dogcatcher failed to communicate his acceptance of the offer to the woman.

(D) Noncommunication of acceptance would be least helpful because the contract with the woman was unilateral and did not require a notice of acceptance; rather, performance was sufficient. (A) is helpful because the fact that the dogcatcher was already under contract might have impact with respect to the preexisting duty rule. (B) is helpful because the fact that the dogcatcher had been compensated already would not have a bearing on a contract theory but might on a quasi-contractual theory. (C) is helpful because the publication of the revocation in the local newspaper may have effectively revoked the woman's offer.

Which one of the following contracts involving an interest in land does not come within the Statute of Frauds? AA contract to buy and sell real estate and divide the profits BA mortgage contract CA contract forming an easement of more than one year DA contract for a lease of real property for more than one year

A A contract to buy and sell real estate and divide the profits does not come within the Statute of Frauds even though the end result may be an interest in land. A contract creating an easement of more than one year is an interest in land covered by the Statute of Frauds. A mortgage contract is an interest in land covered by the Statute of Frauds. A contract for a lease of real property for more than one year is an interest in land covered by the Statute of Frauds.

An art collector was interested in buying a painting from his neighbor. The neighbor told the collector that he could have the painting for $30,000. The collector wanted to think the purchase over. Therefore, the two agreed in writing that the neighbor would keep the offer open for 30 days in exchange for $500, which the collector paid. The terms of the written agreement provided that the offer would expire at 11:59 p.m. on September 30 if the collector failed to accept by that time. On September 20, the collector telephoned his neighbor and told him, "The more I think about it, the less I think that I want your painting." The neighbor responded, "That's your decision to make." On September 26, one of the neighbor's friends was visiting him, saw the painting, and offered his friend (the neighbor) $35,000 for it. On September 27, the neighbor mailed a $50 check to the collector with a letter stating that he was terminating his offer to the collector regarding the painting and refunding 10% of the money that the collector paid him to keep the offer open. He mailed the letter at 11:59 p.m. on September 27. The collector received the letter at 11:30 a.m. on September 29. On September 28, at 9:30 a.m., the collector mailed a letter to his neighbor stating that he had decided to purchase the painting and a certified check in the amount of $30,000 was enclosed. Two hours later, the neighbor sold the painting to his friend for $35,000. The neighbor received the collector's letter on October 1 and immediately mailed the check back to the collector. Can the collector maintain a successful legal action against his neighbor? (A) Yes, because the neighbor sold the painting after the collector's effective acceptance, and before the neighbor's revocation became effective. (B) Yes, because in his revocation the neighbor did not refund the full $500 to the collector. (C) No, because the neighbor effectively revoked his offer before the collector accepted. (D) No, because the collector's power to accept lapsed before he effectively accepted.

(D) The collector's power to accept lapsed because the option contract specified that the offer would expire at 11:59 p.m. on September 30. Hence, the power had to be exercised prior to that time and it was not. The mailbox rule does not apply to the exercise of options. In such cases, acceptance is effective when received by the offeror, here on October 1. Thus, (D) is correct. (A) is wrong because, for the reasons discussed above, the collector did not effectively accept before his option expired. (C) is wrong for two reasons: (i) a revocation is not effective until received; and (ii) because the contract is an option, the offeror's power to terminate the offer through revocation is Contracts Answers 7. limited. Even if the revocation had arrived earlier, the neighbor lacked the power to revoke. (B) is irrelevant. Returning the consideration, in and of itself, would not give the offeror the power to revoke in an option situation.

On February 1, the owner of a bowling alley read in a magazine an ad from a major manufacturer of bowling balls offering sets of 40 balls in various weights and drilled in various sizes for $10 per ball. The owner immediately filled out the order form included in the ad for the 40 balls and deposited it, properly stamped and addressed, into the mail. On February 2, the bowling alley owner received in the mail a letter from the manufacturer, sent out as part of its advertising campaign, stating in relevant part that it will sell the bowling alley owner 40 bowling balls at $10 per ball. A day later, on February 3, the manufacturer received the bowling alley owner's order. On February 4, the balls were shipped. On what day did an enforceable contract arise? (A) February 1, the day the bowling alley owner deposited his order in the mail. (B) February 2, the day the bowling alley owner received the letter from the manufacturer. (C) February 3, the day the manufacturer received the bowling alley owner's letter. (D) February 4, the day the balls were shipped.

(D) The contract arose when the balls were shipped. The general rule is that an offer can be accepted by performance or a promise to perform unless the offer clearly limits the method of acceptance. Here, the offer would be the bowling alley owner's order, because a magazine ad is usually held to be merely solicitation to accept offers rather than an offer. Thus, the manufacturer accepted and the contract was formed when it shipped the balls. (A) is wrong because the bowling alley owner's order was an offer to buy, and no contract could be formed until that offer was accepted. (B) is wrong because this is a case of crossing offers; even though both offers contain the same terms, they do not form a contract. (C) is wrong because no contract will be formed until there has been an acceptance, and, as stated, the bowling alley owner's letter was merely an offer.

A man borrowed $5,000 from his colleague to purchase stock and agreed in writing to repay the loan on or before August 1. On August 1, the man notified his colleague that he would be unable to pay back the $5,000. He told her that he could send her a check for $2,500 and that, in addition, he could give her an antique diamond ring that had been recently appraised at $2,200. The colleague liked the ring and agreed to accept it plus $2,500 in cash as payment for the loan. On August 2, a courier delivered the ring and a certified check for $2,500 to the colleague. She took the check but told the courier to return the ring to the man. The man received the ring back the same afternoon. Meanwhile, the colleague deposited the check in her bank, and the next day filed suit against the man for $2,500. The man consulted an attorney as to whether he has a valid defense against his colleague's suit. Assuming there are no Statute of Frauds issues, what advice should the attorney give the man? (A) The man has no defense against his colleague's suit, because the amount of the debt was undisputed. (B) The man has no defense against his colleague's suit, because she properly exercised her right to enforce the original agreement by refusing tender of the ring. (C) The man has no defense at law, but he may successfully defend in equity under a specific performance theory because the ring is unique. (D) The man has the option of defending in equity under a specific performance theory or waiting until his colleague obtains a judgment against him and then suing her for breach.

(D) The man has the option of defending in equity under a specific performance theory or waiting until his colleague obtains a judgment against him and then suing her for breach. He has either option available because his colleague is in breach of their accord agreement. An accord is an agreement in which one party to an existing contract agrees to accept, in lieu of the performance that she is supposed to receive from the other party, some other, different performance. The accord must be supported by consideration, but the consideration is sufficient if it is of a different type than called for under the original contract, even if the substituted consideration is of less value. An accord suspends the right to enforce the original agreement. Performance of the accord (i.e., satisfaction) cuts off the parties' rights to enforce the original contract and discharges the accord. Here, the accord was supported by sufficient consideration because the man was giving a ring in lieu of some cash. The man's duties under the accord were discharged when he timely tendered delivery of the ring and cash. By refusing the ring and filing suit for the part of the original debt that has not been paid, the colleague has breached the accord agreement. If a creditor breaches an accord agreement, the debtor has the option of either raising the accord agreement as an equitable defense in the creditor's action and asking that it be dismissed, or waiting until he is damaged (i.e., until the creditor is successful in an action on the original contract) and then bringing an action at law for damages for breach of the accord contract. (A) is incorrect because the amount of the debt does not have to be in dispute to have an enforceable accord, as long as there was some alteration in the debtor's consideration, as discussed above. (B) is incorrect because the colleague would have the right to enforce the original contract only if the man had breached the accord agreement. Here, the man's tender of the ring discharged his duty under the accord agreement, precluding his colleague from suing on the original contract. (C) is incorrect because the man has both a breach of contract remedy and an equitable defense option available to him. Also, whether the ring is unique does not affect his right to specific performance of the accord agreement; he is simply raising the agreement as an equitable defense to prevent the colleague from continuing with her suit on the original contract.

The owner of an apartment building contracted with a painter to paint the porches of the apartments for $5,000. The contract was specifically made subject to the owner's good faith approval of the work. The painter finished painting the porches. The owner inspected the porches and believed in good faith that the painter had done a bad job. The painter demanded payment, but the owner told him that the paint job was poor and refused to pay. The painter pleaded that he was desperately in need of money. The owner told the painter that she would pay him $4,500, provided he repainted the porches. The painter reluctantly agreed, and the owner gave the painter a check in the amount of $4,500. The painter went to his bank, indorsed the check "under protest" and signed his name, then deposited the check in his account. He never returned to repaint the porches. The painter sues the owner for $500, which he believes is still owed to him on his contract to paint the porches. Will he prevail? (A) Yes, because he indorsed the check "under protest." (B) Yes, but only if he repaints the porches. (C) Yes, because he performed the contract by painting the porches the first time. (D) No, even if he repaints the porches.

(D) The painter will be unable to recover the $500 because he did not satisfy the condition precedent to payment under the contract. A party does not have a duty to perform if a condition precedent to that performance has not been met. Here, the parties made the owner's satisfaction with the painter's paint job a condition precedent to the owner's duty to pay the $5,000. Because the owner was not satisfied with the paint job, her duty to pay the painter never arose. The fact that the owner offered to give the painter $4,500 if he repainted the porches has no effect on this analysis, because the offer constituted a new contract, the owner having been excused from the old one. (A) is wrong because it does not matter whether the painter indorsed under protest. The indorsement will not change the result here because the new contract did not seek to discharge any contractual duty—the owner was already excused from her duties because the condition precedent was never met. (B) is wrong because the old contract, which provided for payment of $5,000, is considered to be at an end. Under the terms of the new contract, the painter is entitled to only $4,500, provided he repaints the porches. (C) is wrong because the condition precedent to the payment of $5,000, the owner's satisfaction, was not met. The courts have held such conditions to be valid—not illusory promises—because of the promisor's duty to exercise good faith in 8. Contracts Answers assessing satisfaction. Here, the facts state that the owner believed in good faith that the painter had done a bad job; thus, the painter is not entitled to payment under the original contract. Note that since he has not performed under the new contract, he is in breach and not entitled to the $4,500 already paid.

The owner of a stationary bicycle wrote a letter to her friend offering to sell her stationary bicycle to him for $150. The friend received the letter on January 18. On January 19, he mailed a letter back saying that he was not interested in purchasing the bike because he had just purchased a gym membership. However, the friend changed his mind the next day and mailed a letter to the owner accepting her offer to sell the bicycle and enclosing a certified check for $150. The owner received the friend's rejection letter on January 21 but put it aside without reading it. The next day, she received the friend's acceptance letter, which she opened and read immediately. Do the parties have a contract? (A) Yes, because under the mailbox rule an acceptance is effective on dispatch, while a rejection is effective on receipt. (B) Yes, because the friend paid for the bicycle when he accepted the offer to buy it. (C) No, because the acceptance was dispatched after the rejection. (D) No, because the mailbox rule does not apply—whichever is received first controls.

(D) The parties do not have a contract, because the mailbox rule does not apply when the offeree sends a rejection, followed by an acceptance. In such a case, whichever is received first controls. Contracts Answers 5. Under the mailbox rule, acceptance by mail or similar means creates a contract at the moment of posting, with a couple of exceptions not relevant here. Rejection, on the other hand, is effective when received. So, if the mailbox rule had applied, there would have been a contract, because the friend's acceptance was mailed before his rejection letter was received. But because the mailbox rule does not apply here, and the matter is decided based on which letter was received first, there is no contract, because the friend's rejection letter was received by the bicycle owner a day before his acceptance letter was received by her. (A) is incorrect because, as discussed above, the mailbox rule does not apply when a rejection is sent before an acceptance; rather, whichever is received first controls. The fact that the bicycle owner did not read the rejection does not matter; it still was received by her before the acceptance. [See Restatement (Second) Contracts §68] (B) is incorrect because whether the friend paid for the bicycle is irrelevant. He sent the certified check (and his acceptance) after he sent his rejection, and the rejection was received first. (C) is incorrect because when a rejection by mail is followed by an acceptance by mail, the rule is that whichever is received first controls, not whichever is dispatched first. Thus, although it is true that there is no contract between the parties, it is because the friend's rejection letter was received by the bicycle owner first, rather than because it was mailed first.

On April 1, a music store owner offered to sell a rare piano to his best customer, a concert pianist, for $100,000. The following day, the pianist, who performs around the world with two of the several pianos he has purchased from the store, wrote to the store owner: "I have decided to purchase the piano. A check for $100,000 is enclosed. I am leaving in one week for Canada. I will be gone for one month and will pick up the piano when I return. I will pay you to store the piano in your air-conditioned warehouse." One week later, the pianist left for Canada without hearing from the music store owner. What does the letter that the pianist wrote to the store owner constitute? (A) A conditional acceptance. (B) A rejection of the offer. (C) An acceptance, and the store owner is not bound to store the piano. (D) An acceptance, and the store owner must store the piano but is entitled to the reasonable value of that service.

(D) The pianist's letter to the store owner is an acceptance, and the store owner must store the piano. This is a contract for a sale of goods and thus is governed by the UCC. Under the UCC, an acceptance with additional terms does not constitute a rejection and counteroffer, but rather is an effective acceptance unless made expressly conditional on the assent to the additional terms. Here, the pianist expressly accepted the store owner's offer and included an additional term adding one month of storage. The acceptance was not expressly conditional on the store owner's assent to the storage term. Thus, the acceptance was sufficient to create a contract. Whether additional terms become part of the agreement depends on whether both parties are merchants. If both parties are merchants, the additional terms become part of the contract unless they materially alter the terms of the offer, the offer expressly limited the acceptance to its terms, or they are objected to within a reasonable time. A "merchant" is one who regularly deals in goods of the kind sold or 6. Contracts Answers who otherwise by his occupation holds himself out as having knowledge or skill peculiar to the practices or goods involved. For purposes of the UCC battle of the forms provisions, a merchant is almost anyone in business because anyone in business has knowledge of business practices. Here, the store owner is clearly a merchant. The pianist, by virtue of his occupation, has knowledge or skill peculiar to the goods (piano) involved. He plays the piano professionally and has purchased several pianos for this purpose. The additional term included in the pianist's acceptance did not materially alter the terms of the offer (i.e., it did not change a party's risk or remedies), the offer was not limited to its terms, and the facts indicate that the store owner did not object. Thus, the additional term regarding storage becomes part of the agreement. (A) is incorrect because the pianist did not make the acceptance conditional on the owner's assent to the additional term. (B) is incorrect because, unlike at common law, under the UCC, an acceptance that contains additional terms (i.e., one that is not a mirror image of the offer) is not a rejection and counteroffer. (C) would be the correct answer if the pianist were not a merchant. If one of the parties to a contract is not a merchant, any additional terms in the acceptance will be ignored unless specifically accepted.

Valuable consideration must have: ALegal value BEconomic value CSubstantial value

A One of the basic elements of consideration is that which is bargained for must be considered of legal value or, as it is traditionally stated, it must constitute a benefit to the promisor or a detriment to the promisee. The benefit to the promisor need not have economic value. Peace of mind or the gratification of influencing the mind of another may be sufficient to establish bargained-for consideration, provided that the promisee is not already legally obligated to perform the requested act. There is no requirement of substantial value.

A promise to choose among one of several alternative means of performance, only one of which involves an actual legal detriment, will be deemed __________. AValuable consideration if the choice of performance rests with the promisee BValuable consideration if the choice of performance rests with the promisor CIllusory, no matter where the choice of performance rests

A A promise to choose among one of several alternative means of performance, only one of which involves an actual legal detriment, will be deemed valuable consideration so long as the power to choose rests with the promisee or some third party not under the control of the promisor. If the promisee (or the third party) chooses the one alternative that involves a legal detriment, the promisor is bound to perform; thus, his promise is not illusory. If the power to choose rests with the promisor, such a promise will be deemed illusory. An illusory promise is one in which the promisor is not actually bound to perform. The promisor could simply choose the alternative with no legal detriment. Consideration fails in such instances because the agreement lacks mutuality. Mutuality requires that consideration exists on both sides of the contract.

What is an "illusory" promise? AA promise where the promisor is not bound to perform BA promise where the promisor's performance is conditioned on the promisor's satisfaction CA promise where the promisor has some choice or discretion in the means of performance

A An illusory promise is one in which the promisor is not bound to perform. Consideration fails in such instances because the agreement lacks mutuality. Mutuality requires that consideration exists on both sides of the contract. A promise is not illusory just because the promisor has some choice or discretion in the means of performance, or because performance is conditioned on the promisor's satisfaction. The promisor still has a duty to perform in good faith in such situations. Since both the promisor and the promisee are bound to perform, mutuality exists.

A conditional promise is unenforceable if: AThe condition is entirely within the promisor's control BThe condition is the promisor's satisfaction with the performance CThe condition is entirely within the promisee's control DThe condition is extremely remote

A Conditional promises are enforceable unless the condition is entirely within the promisor's control. Such a promise will be deemed illusory. An illusory promise is one in which the promisor is not actually bound to perform. The promisor could simply choose to assert his control over the condition so that he suffers no legal detriment. Consideration fails in such an instance because the agreement lacks mutuality. Mutuality requires that consideration exists on both sides of the contract. Conditional promises are enforceable no matter how remote the contingency. Promises conditioned on the promisor's satisfaction are enforceable because the party has a duty to act in good faith. If the condition is entirely within the promisee's (or some third party's) control, the promise is enforceable as long as it involves a possibility of legal detriment, no matter how remote. The promisee or the third party could assert control over the condition such that the promisor is obligated to perform; thus the promisor's promise is not illusory.

Conditional promises are generally enforceable, unless the __________. Acondition is entirely within the promisor's control Bcontingency is too remote Cpromise is conditioned on one party's satisfaction Dcondition is based on a subjective standard

A Conditional promises are enforceable, no matter how remote the contingency, unless the "condition" is entirely within the promisor's control. Furthermore, a condition may be based on a subjective standard, such as one party's satisfaction with the goods.

In judging the validity of consideration, courts of law __________ inquire into the adequacy of the consideration. AWill not BWill CMay or may not

A Courts of law normally will not inquire into the adequacy of consideration, such as when one party wishes to contract to sell an item of high market value for a relatively low price. In contrast, courts of equity may consider the relative values of the consideration and deny an equitable remedy if they find a contract to be unconscionable.

In a contract in which the seller is in Michigan and the buyer is in Texas, which of the following terms is interpreted as creating a destination contract? AF.O.B. the buyer's place of business BF.A.S. Texas CF.O.B. Michigan DF.A.S. the buyer's place of business

A F.O.B. the buyer's place of business is a destination contract. Under an F.O.B. (free on board) contract, the seller has the risk of loss until the goods reach the location specified. An F.O.B. buyer's place of business is a destination contract, so the seller has the risk of loss until the goods reach the buyer's place of business. F.A.S. stands for "free alongside." The term is generally used only when goods are to be shipped by boat. The risk of loss passes to the buyer once the goods are delivered to the dock. Because the risk of loss shifts when the goods are delivered to the carrier at the dock, these are shipment contracts. F.O.B. Michigan is the same as F.O.B. the seller's location. Because the seller does not bear the risk of loss while the goods are in transit, this is considered a shipment contract.

The promise to refrain from suing on a claim may constitute consideration provided that __________. Athe claimant reasonably and in good faith believes his claim to be valid Bthe claimant has a substantial likelihood of success on his claim Cthe claim is timely Dthe claim is for $500 or more

A If the claimant reasonably and in good faith believes his claim to be valid, forbearance of the legal right to have his claim adjudicated constitutes detriment and consideration. There is no dollar threshold on the amount of the claim; thus, it need not be for $500 or more. Whether the claim is actually invalid or timely, and whether the claimant has a substantial likelihood of success on his claim are not factors in determining whether the forebearance is sufficient consideration

A 17-year-old contracts to buy his neighbor's car. The neighbor is 25 years old. The agreement between the two parties is: AVoidable by the minor but not by the neighbor BBinding as to both parties CVoidable by the neighbor but not by the minor DVoidable by both parties

A In a contract between a minor and an adult, the contract is binding as to the adult and voidable as to the minor. Minors under the age of 18 (also called infants) lack capacity to enter into a contract binding on themselves. Adults have the capacity to bind themselves under a contract. Thus, a contract entered into between a minor and an adult is voidable by the minor but binding on the adult.

A contract entered into between an infant and an adult is __________. Avoidable by the infant but binding on the adult Bvoidable by either party Cvoid Dvoidable by the adult but binding on the infant

A Infants (generally those younger than age 18) lack capacity to enter into a contract binding on themselves. Adults have the capacity to bind themselves under a contract. Thus, a contract entered into between an infant and an adult is voidable by the infant but binding on the adult.

A pedestrian shoved a child out of the path of a speeding car and in doing so sustained significant injuries. The child's grateful parents promised to pay the pedestrian's medical bills but then later refused to pay. Which one of the following offers the pedestrian's best hope of recovery against the child's parents for the cost of the medical bills? AThe material benefit rule BThe preexisting legal duty rule CThe moral consideration rule

A Out of these choices, the pedestrian's best hope of recovery is found in the material benefit rule. Under a modern trend, some courts will enforce a promise if: (i) it is based on a material benefit that was previously conferred by the promisee on the promisor, and (ii) the promisee did not intend to confer the benefit as a gift. This includes situations, such as this one, in which the promisee performed an unrequested act during an emergency. The pedestrian will not be able to recover based on moral consideration. The general rule is that if an act was performed before the promise was made, it will not satisfy the bargain requirement. The preexisting legal duty rule is not applicable to these facts. That rule states that a promise to perform, or the performance of, an existing legal duty is not consideration. There was no preexisting duty here.

Which of the following is not a general rule of contract construction? APrinted provisions will prevail over handwritten provisions BCustom and usage in the particular business and in the particular locale where the contract is either made or to be performed is considered CAmbiguities in a contract are construed against the party preparing the contract DWords will be construed according to their "ordinary" meaning

A Printed provisions will not prevail over handwritten provisions. In fact, if provisions are inconsistent, written or typed provisions will prevail over printed provisions (which indicate a form contract). Courts construe words according to their "ordinary" meaning unless it is clearly shown that they were meant to be used in a technical sense. Courts generally look to see what custom and usage is in the particular business and in the particular locale where the contract is either made or is to be performed. Ambiguities in a contract are construed against the party preparing the contract, absent evidence of the intention of the parties

Which of the following promises is commonly considered to be illusory? AA promise with an unqualified right to cancel or withdraw at any time BA promise conditioned on the promisor's satisfaction CA promise to purchase all that one requires DA promise to sell all that one decides to make

A Reservation of an unqualified right to cancel or withdraw at any time would be considered an illusory promise. "Requirements" contracts (i.e., promises to purchase all that one requires) and "output" contracts (i.e., promises to sell all that one decides to make) are enforceable, as the promisor has parted with the legal right to buy (or sell) the goods he may need (or make) from (or to) another source. A promise conditioned on the promisor's satisfaction is not illusory because the promisor is constrained by good faith (for contracts involving personal taste) and a reasonable person standard (for contracts involving mechanical fitness, utility, or marketability).

A service contract that by its terms __________ is subject to the Statute of Frauds. ACannot be performed within one year BMight extend to longer than one year CIs for the lifetime of one of the parties

A The Statute of Frauds requires that certain contracts be evidenced by a writing signed by the parties sought to be bound. A service contract that by its terms cannot be performed within one year is subject to the Statute of Frauds. The date runs from the date of the agreement and not from the date of performance. If the contract might extend to longer than one year, but it is still possible to complete within one year, it is not within the one-year prong of the Statute of Frauds, even though actual performance may extend beyond the one-year period. A contract for the lifetime of one of the parties is not within the Statute because it is capable of performance within a year, since a person can die at any time. A. INTRODUCTIONEven if an agreement is supported by valuable consideration or a recognized substitute, contract rights may still be unenforceable because there is a defense to formation of the contract, because there is a defect in capacity (making the obligations voidable by one of the parties), or because a defense to enforcement of certain terms exists.B. ABSENCE OF MUTUAL ASSENT 1. Mutual Mistake as to Existing FactsIf both parties entering into a contract are mistaken about existing facts (not future happenings) relating to the agreement, the contract may be voidable by the adversely affected party if:(i) The mistake concerns a basic assumption on which the contract is made (e.g., the parties think they are contracting for the sale of a diamond but in reality the stone is a cubic zirconia));(ii) The mistake has a material effect on the agreed-upon exchange (e.g., the cubic zirconia is worth only a hundredth of what a diamond is worth)); and(iii) The party seeking avoidance did not assume the risk of the mistake. a. Not a Defense If Party Bore the RiskMutual mistake is not a defense if the party asserting mistake as a defense bore the risk that the assumption was mistaken. This commonly occurs when one party is in a position to better know the risks than the other party (e.g., contractor vs. homeowner) or where the parties knew that their assumption was doubtful (i.e., when the parties were consciously aware of their ignorance). 1) Mistake in Value Generally Not a DefenseIf the parties to a contract make assumptions as to the value of the subject matter, mistakes in those assumptions will generally not be remedied - even though the value of the subject matter is generally a basic assumption and the mistake creates a material imbalance - because both parties usually assume the risk that their assumption as to value is wrong. 2. Compare - Unilateral MistakeIf only one of the parties is mistaken about facts relating to the agreement, the mistake will not prevent formation of a contract. However, if the nonmistaken party knew or had reason to know of the mistake made by the other party, the contract is voidable by the mistaken party. As with mutual mistake, the mistake must have a material effect on the agreed-upon exchange and the mistaken party must not have borne the risk of the mistake.EXAM TIP:Unilateral mistakes arise most commonly when one party makes a mechanical error in computation. Whenever you see facts in which a subcontractor's bid was wrong or acreage in a land sale contract was miscalculated, consider whether the contract may be avoided due to unilateral mistake.3. Mistake by the Intermediary (Transmission)When there is a mistake in the transmission of an offer or acceptance by an intermediary, the prevailing view is that the message as transmitted is operative unless the other party knew or should have known of the mistake.4. Misunderstanding-Ambiguous Contract LanguageIf the contract includes a term with at least two possible meanings, the result depends on the parties' awareness of the ambiguity: (i) Neither party aware - no contract unless both parties intended the same meaning);(ii) Both parties aware - no contract unless both parties intended the same meaning);(iii) One party aware - binding contract based on what the ignorant party reasonably believed to be the meaning of ambiguous words.Ambiguity is one area where subjective intent is taken into account. 5. Misrepresentation a. Fraudulent Misrepresentation (Fraud in the Inducement)If a party induces another to enter into a contract by using fraudulent misrepresentation (i.e., by asserting information she knows is untrue), the contract is voidable by the innocent party if she justifiably relied on the fraudulent misrepresentation. This is fraud in the inducement.b. Material MisrepresentationWhether or not a misrepresentation is fraudulent, the contract is voidable by the innocent party if the innocent party justifiably relied on the misrepresentation and the misrepresentation was material. A misrepresentation is material if: (i) it would induce a reasonable person to agree, or (ii) the maker knows that for some special reason it is likely to induce the particular person to agree, even if a reasonable person would not.EXAM TIP:Keep in mind that a fraudulent misrepresentation need not be spoken or written); it can be inferred from conduct. Concealing a fact, frustrating investigation of a fact, or falsely denying knowledge of a fact is the same as asserting the fact does not exist. However, nondisclosure of a fact is not misrepresentation unless it is material or fraudulent (e.g., false denial of knowledge of a material fact).c. Justified RelianceA party is not entitled to relief if the reliance was unreasonable under the circumstances. However, just because a misrepresentation could have been revealed by the exercise of reasonable care does not mean that reliance was unjustified. Failure to read a contract or use care in reading it does not necessarily preclude a party from avoiding a contract for misrepresentation. d. Innocent Party May Rescind Agreement and Recover Damag esNote that the innocent party need not wait until she is sued on the contract, but may take affirmative action in equity to rescind the agreement. In addition, she may pursue all remedies available for breach of contract (see VIII., infra). C. ABSENCE OF CONSIDERATIONIf the promises exchanged at the formation stage lack the elements of bargain or legal detriment, no contract exists. In this situation, one of the promises is always illusory.D. PUBLIC POLICY DEFENSES-ILLEGALITYIf the consideration or subject matter of a contract is illegal (e.g., a contract to commit a murder), the contract is void. Exceptions: (i) the plaintiff is unaware of the illegality while the defendant knows of the illegality); (ii) the parties are not in pari delicto (i.e., one party is not as culpable as the other)); or (iii) the illegality is the failure to obtain a license when the license is for revenue-raising purposes rather than for protection of the public. If only the purpose behind the contract is illegal, the contract is voidable by a party who was (i) unaware of the purpose); or (ii) aware but did not facilitate the purpose and the purpose does not involve serious moral turpitude.E. DEFENSES BASED ON LACK OF CAPACITY 1. Legal Incapacity to Contract a. Contracts of Infants (Minors)Infants (in most jurisdictions, persons under the age of 18) generally lack capacity to enter into a contract binding on themselves. However, contractual promises of an adult made to an infant are binding on the adult. 1) DisaffirmanceAn infant may choose to disaffirm a contract any time before (or shortly after) reaching the age of majority. If an infant chooses to disaffirm, she must return anything that she received under the contract that still remains at the time of disaffirmance. However, there is no obligation to return any part of the consideration that has been squandered, wasted, or negligently destroyed. a) ExceptionsStates have created a few statutory exceptions for student loans, insurance contracts, and agreements not to reveal an employer's proprietary information.b) Necessaries"Necessaries" are items necessary for the minor's subsistence, health, or education (e.g., food, shelter, clothing, medical care). A minor may disaffirm a contract for necessaries but in most states will be liable in restitution for the value of benefits received. 2) Affirmance upon Attaining MajorityAn infant may affirm, i.e., choose to be bound by his contract, upon reaching majority. He affirms either expressly or by conduct (e.g., by failing to disaffirm the contract within a reasonable time after reaching majority). b. Mental IncapacityOne whose mental capacity is so deficient that he is incapable of understanding the nature and significance of a contract may disaffirm when lucid or by his legal representative. He may likewise affirm during a lucid interval or upon complete recovery, even without formal restoration by judicial action. In other words, the contract is voidable. As in the case of infants, mentally incompetent persons are liable in quasi-contract for necessaries furnished to them.c. Intoxicated PersonsOne who is so intoxicated that he does not understand the nature and significance of his promise may be held to have made only a voidable promise if the other party had reason to know of the intoxication. The intoxicated person may affirm the contract upon recovery. Once again, there may be quasi-contractual recovery for necessaries furnished during the period of incapacity. 2. Duress and Undue InfluenceContracts induced by duress or undue influence are voidable and may be rescinded as long as not affirmed. The common type of duress occurs when a party's assent is procured by an improper threat (e.g., "sign the contract or I'll break your legs"). Generally, taking advantage of another person's economic needs is not duress. However, withholding something someone wants or needs will constitute economic duress if: (i) the party threatens to commit a wrongful act that would seriously threaten the other contracting party's property or finances); and (ii) there are no adequate means available to prevent the threatened loss. Elements of undue influence are: (i) undue susceptibility to pressure by one party, and (ii) excessive pressure by the other party. Undue influence concerns often arise when the dominant party is in a confidential or caregiver relationship with the influenced party. F. STATUTE OF FRAUDSIn most instances, an oral contract is valid. However, certain agreements, by statute, must be evidenced by a writing signed by the party sought to be bound. 1. Writing RequirementThe Statute of Frauds does not require a formal written contract. Among other things, the writing can be a receipt, a letter, a check with details in the memo line, or a written offer that was accepted orally. The Statute requires only one or more writings that: (i) reasonably identify the subject matter of the contract, (ii) indicate that a contract has been made between the parties, and (iii) state with reasonable certainty the essential terms. a. Electronic Record Satisfies Writing RequirementIf a law requires a record to be in writing, an electronic record (e.g., an e-mail) satisfies the law.b. Essential or Material TermsThere is no definitive list of essential terms. What is essential depends on the agreement, its context, and the subsequent conduct of the parties, including the dispute that has arisen. There must be enough in the writing to enable a court to enforce the contract. If an essential term is contained in the writing, evidence is admissible to explain the particulars, but evidence will not be admitted to add a missing term. Examples of essential terms include: identity of the parties, description of the subject matter, and the terms necessary to make the contract definite. Writings evidencing land sale contracts must contain a description of the land and the price, and those for employment contracts must state the length of employment. For the sale of goods, the UCC requires only some signed writing indicating that a contract has been made and specifying the quantity term. EXAM TIPRemember, to be sufficient under the Statute of Frauds, the writing need not be a full-fledged contract, nor need it even be one piece of paper. Thus, several pieces of correspondence (including electronic correspondence) between the parties could be sufficient memoranda of the agreement); a fax or a memo written on a napkin also could suffice. The key is that there be something in writing evidencing the essential terms. 2. Signature RequirementThe signature requirement is liberally construed by most courts. A signature is any mark or symbol made with the intention to authenticate the writing as that of the signer. It need not be handwritten); it can be printed or typed. Under the UCC, a party's initials or letterhead may also be sufficient. An electronic signature is also sufficient.EXAM TIPNote that the memorandum need not be signed by both parties. Only the party to be charged (i.e., the person to be sued) must sign. Thus, in a suit by the buyer against the seller, an otherwise sufficient writing that is signed by the seller but not the buyer satisfies the Statute of Frauds. However, if the seller were suing the buyer, the writing would not be sufficient. (Although there is an exception in contracts for the sale of goods in the case of a merchant's confirmatory memo); see 3.f.2), infra.)3. Agreements Covered a. Executor or Administrator Promises Personally to Pay Estate DebtsA promise by an executor or administrator to pay the estate's debts out of his own funds must be evidenced by a writing.b. Promises to Pay Debt of Another (Suretyship Promises)A promise to answer for the debt or default of another must be evidenced by a writing. The promise may arise as a result of a tort or contract, but it must be collateral to another person's promise to pay, and not a primary promise to pay. However, if the main purpose or leading object of the promisor is to serve a pecuniary interest of his own, the contract is not within the Statute of Frauds even though the effect is still to pay the debt of another (e.g., homeowner promises to pay contractor's debt to building supplier if contractor does not pay, so contractor can obtain supplies to work on homeowner's house).c. Promises in Consideration of MarriageA promise the consideration for which is marriage must be evidenced by a writing. This applies to promises that induce marriage by offering something of value (other than a return promise to marry - e.g., "if you marry my son, I will give the two of you a house").d. Interest in LandA promise creating an interest in land must be evidenced by a writing. This includes not only agreements for the sale of real property, but also:(i) Leases for more than one year);(ii) Easements of more than one year);(iii) Mortgages and most other security liens);(iv) Fixtures); and(v) Minerals (or the like) or structures if they are to be severed by the buyer. 1) Items That Do Not Create an Interest in LandContracts to build a building or to find a buyer for a seller (e.g., a broker's contract) do not create an interest in land. 2) Effect of Performance on ContractsFull performance by the seller will take the contract out of the Statute of Frauds. Part performance by the buyer may also remove the contract from the Statute. (See 5.a.1), infra.) e. Performance Not Within One Year from Date of ContractA promise that by its terms cannot be performed within one year is subject to the Statute of Frauds. Part performance does not satisfy the Statute of Frauds in this case. The date runs from the date of the agreement and not from the date of performance. Note that, even if the contract cannot be performed within a year, full performance by one party will remove it from the Statute.EXAM TIPWatch for a contract measured by a lifetime. A promise to "employ until I die" or "work until I die" is not within the Statute because it is capable of performance within a year - a person can die at any time.f. Goods Priced at $500 or MoreA contract for the sale of goods for a price of $500 or more is within the Statute of Frauds and generally must be evidenced by a signed writing to be enforceable. Note that a writing is sufficient even though it omits or incorrectly states a term, but the contract is not enforceable beyond the quantity of goods shown in the writing. 1) When Writing Not RequiredThere are three situations in which contracts are enforceable without the writing described above: a) Specially Manufactured GoodsIf goods are to be specially manufactured for the buyer and are not suitable for sale to others by the seller in the ordinary course of his business, the contract is enforceable if the seller has, under circumstances that reasonably indicate that the goods are for the buyer, made a substantial beginning in their manufacture or commitments for their purchase before notice of repudiation is received.b) Admissions in Pleadings or CourtIf the party against whom enforcement is sought admits in pleadings, testimony, or otherwise in court that the contract for sale was made, the contract is enforceable without a writing (but in such a case the contract is not enforced beyond the quantity of goods admitted).c) Payment or Delivery of GoodsIf goods are either received and accepted or paid for, the contract is enforceable. However, the contract is not enforceable beyond the quantity of goods accepted or paid for. Thus, if only some of the goods called for in the oral contract are accepted or paid for, the contract is only partially enforceable. If an indivisible item is partially paid for, most courts hold that the Statute of Frauds is satisfied for the whole item. 2) Merchants - Confirmatory Memo RuleIn contracts between merchants, if one party, within a reasonable time after an oral agreement has been made, sends to the other party a written confirmation of the understanding that is sufficient under the Statute of Frauds to bind the sender, it will also bind the recipient if: (i) he has reason to know of the confirmation's contents); and (ii) he does not object to it in writing within 10 days of receipt. EXAM TIPTo determine whether the Statute of Frauds is satisfied (and the contract is enforceable), look carefully for a writing signed by the party to be charged (i.e., sued). If only one party signed the writing, first check to see if the signature is of the party being sued. If not, consider whether the merchants' confirmatory memo rule applies. Be sure that the contract is between merchants); if not, that rule does not apply, and the signature of one party cannot bind the other.EXAM TIPA mnemonic for remembering when a writing signed by the party to be charged is not required for a sale of goods, even if for $500 or more, is SWAP: Specially made goods, Written confirmation by a merchant, Admission in court, or Performance. These facts take the contract out of the Statute of Frauds. 4. Effect of Noncompliance with the StatuteNoncompliance with the Statute of Frauds renders the contract unenforceable at the option of the party to be charged (i.e., the party being charged may raise the lack of a sufficient writing as an affirmative defense). If the Statute is not raised as a defense, it is waived. 5. Situations in Which the Contract Is Removed from the Statute of Frauds a. Performance 1) Land Sale ContractsIf a seller conveys to the buyer (i.e., fully performs), he can enforce the buyer's oral promise to pay. Likewise, the buyer may seek to specifically enforce an oral land sale contract under the doctrine of part performance. Part performance that unequivocally indicates that the parties have contracted for the sale of land takes the contract out of the Statute of Frauds. Most jurisdictions require at least two of the following: payment (in whole or in part), possession, and/or valuable improvements. a) Specific Performance OnlyA purchaser of an interest in land may enforce an oral contract in this manner only in equity (i.e., he may sue only for specific performance, not damages).EXAM TIPWatch for a fact pattern where the parties orally agree to an installment land contract. In the absence of other facts, such as a large down payment, possession plus payment does not unequivocally indicate a contract for the sale of land. Those facts are also consistent with a lease); thus, the purchaser cannot enforce the contract. 2) Sale of Goods ContractsPart performance takes a sale of goods contract out of the Statute of Frauds when: (i) the goods have been specially manufactured, or (ii) the goods have been either paid for or accepted. If a sales contract is only partially paid for or accepted, the contract is enforceable only to the extent of the partial payment or acceptance.3) Services Contracts - Full Performance RequiredAs noted above, an oral contract that cannot be completed within one year but has been fully performed by one party is enforceable. b. Equitable and Promissory EstoppelEstoppel may be applied if it would be inequitable to allow the Statute to defeat a meritorious claim (e.g., defendant falsely and intentionally tells plaintiff that the contract is not within the Statute, defendant induces the plaintiff to change position in reliance on an oral agreement).c. Judicial AdmissionIf a party admits in pleadings or testimony that there is an agreement, it is treated the same as though the party signed a writing. 6. Remedies If Contract Is Within StatuteIf a contract violates the Statute of Frauds, in almost all cases a party can sue for the reasonable value of the services or part performance rendered, or the restitution of any other benefit that has been conferred. (See VIII.C., infra, for a detailed discussion.) EXAM TIPIf the part performance rendered takes the contract out of the Statute of Frauds, the performing party has the option of suing on the contract for expectation damages, rather than merely in restitution for the value of the benefit conferred. EXAM TIPStatute of Frauds issues are often raised in MBE questions. Remember that the Statute does not apply to all contracts. You must check the facts to see whether the contract falls within any of the covered areas (above). An easy way to remember agreements covered by the Statute of Frauds is by using the mnemonic MY LEGS: Marriage,(Within one) Year, L and, Executor (or Administrator),Goods (for $500 or more),Surety. G. UNCONSCIONABILITYThe concept of unconscionability allows a court to refuse to enforce a provision or an entire contract (or to modify the contract) to avoid "unfair" terms, usually due to some unfairness in the bargaining process (i.e., procedural unconscionability). Unfair price alone is not a ground for unconscionability. 1. Common Instances of Procedural Unconscionability a. Inconspicuous Risk-Shifting ProvisionsStandardized printed form contracts often contain a material provision that seeks to shift a risk normally borne by one party to the other. Typically, such clauses are found in the fine print ("boilerplate") in printed form contracts. Courts have invalidated these provisions because they are inconspicuous or incomprehensible to the average person, even if brought to his actual attention.b. Contracts of Adhesion - "Take It or Leave It"Courts will deem a clause unconscionable and unenforceable if the signer is unable to procure necessary goods, such as an automobile, from any seller without agreeing to a similar provision.c. Exculpatory ClausesAn exculpatory clause releasing a contracting party from liability for his own intentional wrongful acts is usually found to be unconscionable because such a clause is against public policy in most states. Exculpatory clauses for negligent acts may be found to be unconscionable if they are inconspicuous (as discussed above), but commonly are upheld if they are in contracts for activities that are known to be hazardous (e.g., a contract releasing a ski hill operator for liability for negligence often will be upheld).d. Limitations on RemediesA contractual clause limiting liability for damages to property generally will not be found to be unconscionable unless it is inconspicuous. However, if a contract limits a party to a certain remedy and that remedy fails of its essential purpose (e.g., the contract limits remedies to repair and the item cannot be repaired), a court may find the limitation unconscionable and ignore it. 2. TimingUnconscionability is determined by the circumstances as they existed at the time the contract was formed.3. Effect If Court Finds Unconscionable ClauseIf a court finds as a matter of law that a contract or any clause of the contract was unconscionable when made, the court may: (i) refuse to enforce the contract); (ii) enforce the remainder of the contract without the unconscionable clause); or (iii) limit the application of any clause so as to avoid an unconscionable result.EXAM TIPUnconscionability is seldom a good defense on the MBE. That a contract turned out badly for one party is insufficient in itself to give rise to unconscionability. Look for great differences in bargaining power (e.g., big company vs. average consumer) before finding a contract or clause is unconscionable.

The __________ is a possible exception to the general rule against the use of "past" consideration as the basis for a contract. AMaterial benefit rule BPreexisting legal duty rule CParol evidence rule

A The material benefit rule is a possible exception to the general rule against the use of past consideration as the basis for a contract. Under the rule, some courts will enforce a promise if: (i) it is based on a material benefit that was previously conferred by the promisee on the promisor, and (ii) the promisee did not intend to confer the benefit as a gift. This includes situations in which the promisee performed an act at the promisor's request or performed an unrequested act during an emergency. The preexisting legal duty rule provides that a promise to perform, or the performance of, an existing legal duty is not consideration. The parol evidence rule is not an exception to the rule regarding past consideration. It states that where the parties to a contract express their agreement in a writing with the intent that it embody the final expression of their bargain, any other expressions—written or oral—made prior to the writing, as well as any oral expressions contemporaneous with the writing, are inadmissible to vary the terms of the writing

Under U.C.C. Article 2, a contract for the sale of goods may be modified without consideration only if: AThe modification is sought in good faith. BIt is a requirements or output contract. CThere is an honest dispute as to the legal duty owed by a party. DUnforeseen circumstances make performance impracticable.

A Under Article 2, all contract modifications sought in good faith are binding without consideration. Both an honest dispute as to the legal duty owed and unforeseen circumstances that make performance impracticable are exceptions to the preexisting legal duty rule. They are examples of adequate consideration for all contracts, not examples of the facts necessary to modify a sales contract without consideration. Requirements and output contracts are treated the same as other sales contracts in terms of modification. As with other sales contracts, no consideration is required for modification if the modification is sought in good faith. A. INTRODUCTIONOnce you have determined that a contract exists, the next thing you must do is determine what its terms are.B. GENERAL RULES OF CONTRACT CONSTRUCTIONThere are a number of general rules of construction applied by the courts when interpreting contracts. The following are among the more frequently invoked: 1. Contracts will be construed as a "whole"); specific clauses will be subordinated to the contract's general intent);2. The courts will construe words according to their "ordinary" meaning unless it is clearly shown that they were meant to be used in a technical sense);3. If provisions appear to be inconsistent, written or typed provisions will prevail over printed provisions);4. It is important to note that the courts generally will try to reach a determination that a contract is valid and enforceable);5. Ambiguities in a contract are construed against the party preparing the contract, absent evidence of the intention of the parties);6. The parties' course of dealing (i.e., the sequence of conduct concerning previous transactions between the parties to a particular transaction that may be regarded as establishing a common basis of their understanding));7. A usage of trade (i.e., a practice or method of dealing, regularly observed in a particular business setting so as to justify an expectation that it will be followed in the transaction in question));8. The parties' course of performance (i.e., if a contract involves repeated occasions for performance by either party and the other party has the opportunity to object to such performance, any course of performance accepted or acquiesced to is relevant in determining the meaning of the contract)); and9. When rules conflict: (i) express terms are given greater weight than course of performance, course of dealing, and usage of trade); (ii) course of performance is given greater weight than course of dealing or usage of trade); and (iii) course of dealing is given greater weight than usage of trade. C. PAROL EVIDENCE RULE - SUPPLEMENTING, EXPLAINING, OR CONTRADICTING TERMSWhen the parties to a contract express their agreement in a writing with the intent that it embody the final expression of their bargain (i.e., the writing is an "integration"). Any other expressions - written or oral - made prior to the writing, as well as any oral expressions contemporaneous with the writing, are inadmissible to vary the terms of the writing. 1. Is the Writing an "Integration"?There are two components: (i) whether the writing was intended as the final expression of the agreement); and (ii) whether the integration was intended to be complete or partial. Evidence is admissible to show the parties' intent. a. Partial Integration - Additional Terms PermittedIf an integration is complete, the writing cannot be contradicted or supplemented. If, however, the integration is partial, the writing may not be contradicted but may be supplemented by proving consistent additional terms. The UCC presumes all writings are partial integrations. b. Effect of Merger ClauseA merger clause recites that the agreement is the complete agreement between the parties. The presence of a merger clause is usually determinative in large commercial contracts. For most contracts, however, the modern trend is to consider it as one factor in determining integration. EXAM TIPA memo prepared by one party and not shown to the other can never be an integration because the parties could not have intended it to be the final complete expression of their agreement when one party has not even seen it. The writing is merely evidence of the agreement. Note that a confirmatory memo may be a partial integration under the UCC because it was sent to the other party and that party was aware of its contents. 2. Evidence Outside Scope of RuleBecause the rule prohibits admissibility only of extrinsic evidence that seeks to vary, contradict, or add to an "integration," other forms of extrinsic evidence may be admitted if they will not bring about this result, i.e., they will fall outside the scope of the parol evidence rule. a. Validity IssuesA party to a written contract can attack the agreement's validity. The party acknowledges (concedes) that the writing reflects the agreement but asserts, most frequently, that the agreement never came into being because of any of the following: 1) Formation DefectsFormation defects (e.g., fraud, duress, mistake, and illegality) may be shown by extrinsic evidence.2) Conditions Precedent to EffectivenessIf a party asserts that there was an oral agreement that the written contract would not become effective until a condition occurred, all evidence of the understanding may be offered and received. b. Collateral Agreements and Naturally Omitted TermsParol evidence is often said to be admissible if the alleged parol agreement is collateral to the written obligation (i.e., related to the subject matter but not part of the primary promise) and does not conflict with it. The Restatements of Contracts include a similar concept with a more definitive approach: the naturally omitted terms doctrine. The doctrine allows evidence of terms that would naturally be omitted from the written agreement. A term would naturally be omitted if: (i) it does not conflict with the written integration); and (ii) it concerns a subject that similarly situated parties would not ordinarily be expected to include in the written instrument.c. InterpretationIf there is uncertainty or ambiguity in the written agreement's terms or a dispute as to the meaning of those terms, parol evidence can be received to aid the fact-finder in reaching a correct interpretation of the agreement. However, if the meaning of the agreement is plain, parol evidence is inadmissible.d. Showing of "True Consideration"The parol evidence rule will not bar extrinsic evidence showing the "true consideration" paid (e.g., evidence that the consideration stated in the contract was never paid).e. ReformationIf a party to a written agreement alleges facts (e.g., mistake) entitling him to reformation of the agreement, the parol evidence rule is inapplicable.f. Subsequent ModificationsParol evidence can be offered to show subsequent modifications of a written contract.g. Additional Terms Under Article 2Article 2 generally follows the rules discussed above, providing that a party cannot contradict a written contract but may add consistent additional terms unless: (i) there is a merger clause, or (ii) the courts find from all of the circumstances that the writing was intended as a complete and exclusive statement of the terms of the agreement. Article 2 also provides that a written contract's terms may be explained or supplemented by evidence of course of performance, course of dealing, and usage of trade-regardless of whether or not the writing appears to be ambiguous. D. ARTICLE 2 PROVISIONS ON INTERPRETING CONTRACTS 1. Supplemental ("Gap Filler") TermsRecall that the key to forming a contract for the sale of goods is the quantity term (see II.B.2.b.1)b), supra). If other terms are missing from the agreement, Article 2 has gap filler provisions to fill in the missing term(s). a. PriceIf: (i) nothing has been said as to price); (ii) the price is left open to be agreed upon by the parties and they fail to agree); or (iii) the price is to be fixed in terms of some standard that is set by a third person or agency and it is not set, then the price is a reasonable price at the time for delivery.b. Place of DeliveryIf the place of delivery is not specified, the place usually is the seller's place of business, if he has one); otherwise, it is the seller's home.c. Time for Shipment or DeliveryIf the time for shipment or delivery is not specified, shipment/delivery is due in a reasonable time.d. Time for PaymentIf the time for payment is not specified, payment is due at the time and place at which the buyer is to receive the goods.e. AssortmentIf a contract provides that an assortment of goods is to be delivered (e.g., blouses in various colors and sizes) and does not specify which party is to choose, the assortment is at the buyer's option. If the party who has the right to specify the assortment does not do so seasonably, the other party is excused from any resulting delay and may either proceed in any reasonable manner (e.g., choose a reasonable assortment) or treat the failure as a breach. 2. Delivery Terms and Risk of LossAll contracts for the sale of goods require delivery of the goods. A contract's delivery terms are important because they determine when risk of loss passes from the seller to the buyer if the goods are damaged or destroyed. a. Noncarrier CaseA noncarrier case is a sale in which it appears that the parties did not intend that the goods would be moved by a common carrier (e.g., when you buy groceries). In such a case, if the seller is a merchant, risk of loss passes to the buyer only when she takes physical possession of the goods. If the seller is not a merchant, risk of loss passes to the buyer upon tender of delivery.b. Carrier CaseA carrier case is a sale in which it appears that the parties intended the goods to be moved by a carrier (e.g., when you order a book from an Internet website). There are two types of carrier cases: shipment contracts and destination contracts. 1) Shipment ContractIf the contract authorizes or requires the seller to ship the goods by carrier but does not require him to deliver them at a particular destination, it is a shipment contract and risk of loss passes to the buyer when the goods are delivered to the carrier. In the absence of a contrary agreement, Article 2 presumes a contract is a shipment contract. a) Seller's Duties Under Shipment ContractIn a shipment contract, the seller must: (i) make a reasonable contract with the carrier on behalf of the buyer); (ii) deliver the goods to the carrier); (iii) promptly notify the buyer of the shipment); and (iv) provide the buyer with any documents needed to take possession of the goods. 2) Destination ContractsIf the contract requires the seller to deliver the goods at a particular destination, the risk of loss passes to the buyer when the goods are tendered to the buyer at the destination.3) Common Delivery Terms a) F.O.B.F.O.B. stands for "free on board." The letters F.O.B. are always followed by a location, and the risk of loss passes to the buyer at the named location. The seller bears the risk and expense of getting the goods to the named location. These contracts can be either shipment contracts or destination contracts, depending on the location named.b) F.A.S.F.A.S. stands for "free alongside." The term is generally used only when goods are to be shipped by boat. Risk of loss passes to the buyer once the goods are delivered to the dock. EXAM TIPAll contracts for goods require an address for delivery. Merely indicating an address for shipment does not make a contract a destination contract. A contract that does not contain an F.O.B. term or any other term explicitly allocating the risk of loss is a shipment contract. CHART c. Effect of Breach on Risk of Loss 1) Defective GoodsIf the buyer has a right to reject the goods, the risk of loss does not pass to the buyer until the defects are cured or she accepts the goods in spite of their defects. Note that a buyer generally has the right to reject for any defect. (See VII.C., infra.)2) Revocation of AcceptanceIf the buyer rightfully revokes acceptance, the risk of loss is treated as having rested on the seller from the beginning to the extent of any deficiency in the buyer's insurance coverage. EXAM TIPBecause of the above rules, if a seller ships nonconforming goods, it eliminates the importance of determining whether a contract is a shipment or destination contract. If the goods are nonconforming, the risk of loss remains on the seller.d. Risk in Sale or Return and Sale on Approval Contracts 1) Sale or ReturnFor the purpose of determining the risk of loss, a sale or return contract (e.g., the buyer takes goods for resale but may return them if she is unable to resell them) is treated as an ordinary sale and the above rules apply. If the goods are returned to the seller, the risk remains on the buyer while the goods are in transit.2) Sale on ApprovalIn a sale on approval (i.e., the buyer takes goods for use but may return them even if they conform to the contract), the risk of loss does not pass to the buyer until she accepts. CHART e. Goods Destroyed Before Risk of Loss PassesIf goods that were identified when the contract was made are destroyed (i) without fault by either party and (ii) before the risk of loss passes to the buyer, the contract is avoided (i.e., the seller's performance is excused). If the goods were not identified until after the contract was made, the seller in this situation would have to prove impracticability (VI.E.4.b., infra) to be discharged. 3. Insurable Interest and IdentificationAs noted above, a buyer often bears the risk of loss before receiving the goods purchased. In order to aid buyers in this situation (and a few others), Article 2 gives buyers a special property interest in goods as soon as they are identified as the ones that will be used to satisfy the contract (e.g., as soon as the seller sets them aside for the buyer). This special property interest is insurable.4. Bilateral Contracts Formed by PerformanceRecall that a contract may be formed by the parties' performance where the mirror image rule is not satisfied and under certain circumstances under Article 2's "battle of the forms" provision. (See II.D.5.b., supra.) In such cases, under Article 2, the contract includes all of the terms on which the writings of both parties agree. Any necessary missing terms are filled in by the supplemental terms provided for in Article 2. a. Compare - Common Law Last Shot RuleThe rule is different in common law contracts. At common law, the contract includes the terms of the last communication sent to the party who performed. 5. WarrantiesContracts for the sale of goods automatically include a warranty of title (in most cases). They also may include certain implied warranties and express warranties. a. Warranty of Title and Against Infringement 1) Warranty of TitleAny seller of goods warrants that the title transferred is good, that the transfer is rightful, and that there are no liens or encumbrances against the title of which the buyer is unaware at the time of contracting. This warranty arises automatically and need not be mentioned in the contract.2) Warranty Against InfringementA merchant seller regularly dealing in goods of the kind sold also automatically warrants that the goods are delivered free of any patent, trademark, copyright, or similar claims. But a buyer who furnishes specifications for the goods to the seller must hold the seller harmless against such claims. b. Implied Warranty of MerchantabilityImplied in every contract for sale by a merchant who deals in goods of the kind sold, there is a warranty that the goods are merchantable. To be merchantable, goods must at least be "fit for the ordinary purposes for which such goods are used." 1) Seller's Knowledge of Defect Not RelevantAs in all implied warranty cases, it makes no difference that the seller himself did not know of the defect or that he could not have discovered it. Implied warranties are not based on negligence but rather on absolute liability that is imposed on certain sellers. c. Implied Warranty of Fitness for a Particular PurposeA warranty will also be implied in a contract for the sale of goods whenever (i) any seller, merchant or not, has reason to know the particular purpose for which the goods are to be used and that the buyer is relying on the seller's skill and judgment to select suitable goods); and (ii) the buyer in fact relies on the seller's skill or judgment.d. Express WarrantiesAny affirmation of fact or promise made by the seller to the buyer, any description of the goods, and any sample or model creates an express warranty if the statement, description, sample, or model is part of the basis of the bargain. For the statement, description, sample, or model to be a part of the basis of the bargain, it need only come at such a time that the buyer could have relied on it when she entered into the contract. The buyer does not need to prove that she actually did rely, although the seller may negate the warranty by proving that the buyer as a matter of fact did not rely. It is not necessary that the seller intended the affirmation of fact, description, model, or sample to create a warranty. 1) Distinguish - Statements of Value or OpinionA statement relating merely to the value of the goods, or a statement purporting to be only the seller's opinion or commendation of the goods, does not create an express warranty. e. Disclaimer of Warranties 1) Warranty of TitleThe title warranty can be disclaimed or modified only by specific language or by circumstances that give the buyer notice that the seller does not claim title or that he is selling only such rights as he or a third party may have (e.g., a sheriff's sale).2) Implied WarrantiesThe implied warranties of merchantability and fitness for a particular purpose can be disclaimed by either specific disclaimers or general methods of disclaimer. a) Specific Disclaimers (1) Disclaimer of Warranty of MerchantabilityThe warranty of merchantability can be specifically disclaimed or modified only by mentioning merchantability. If the sales contract is in writing, the disclaimer must be conspicuous.(2) Disclaimer of Warranty of Fitness for a Particular PurposeThe warranty of fitness for a particular purpose can be specifically disclaimed only by a conspicuous writing. A written disclaimer, according to the statute, is sufficient if it says, for example, "[t]here are no warranties which extend beyond the description on the face hereof."(3) "Conspicuous" DefinedA term is conspicuous when it is "so written, displayed, or presented that a reasonable person against whom it is to operate ought to have noticed it." Language in the body of a writing is conspicuous if: (i) it is in larger type than surrounding text); (ii) it is in a contrasting type, font, or color); or (iii) it is set off from the text by marks that call attention to it. The court, not the jury, decides any fact question as to conspicuousness. b) Other Methods of Disclaiming Implied WarrantiesThe UCC also provides several more general methods for disclaiming implied warranties. (1) By "As Is" or Similar LanguageUnless the circumstances indicate otherwise, the implied warranties of merchantability and fitness can be disclaimed by expressions such as "as is," "with all faults," or other expressions that in common understanding call the buyer's attention to the fact that there are no implied warranties. Although this type of disclaimer does not have to be conspicuous, a hidden or fine-print disclaimer of this type is not effective.(2) By Examination or Refusal to ExamineIf the buyer, before entering into the contract, has examined the goods or a sample or model as fully as she desires or has refused to examine, there is no warranty as to defects that a reasonable examination would have revealed to her.(3) By Course of Dealing, Etc.Implied warranties may also be disclaimed by the course of dealing, course of performance, or usage of trade. EXAM TIPIt may seem odd that there are specific disclaimer methods with detailed requirements, and more general disclaimer methods requiring little formality. In actual practice, it is better to use the specific disclaimers because general disclaimers may be limited by the circumstances. However, on the MBE, an "as is" or "with all faults" disclaimer will generally be as effective as a specific disclaimer. 3) Express WarrantiesThe UCC provides that words or conduct relevant to the creation of express warranties and words or conduct tending to negate such warranties shall wherever possible be construed as consistent with each other, but "negation or limitation is inoperative to the extent that such construction is unreasonable." In other words, once an express warranty is made, it is very difficult to disclaim.4) Limitations on DamagesParties may include in their contract a clause limiting the damages available in the case of breach of warranty (e.g., "remedy for breach of warranty is limited to repair or replacement of the defective goods"). However, such a limitation will not be upheld if it is unconscionable (e.g., causes the remedy to fail of its essential purpose, limits personal injury damages for consumer goods); see IV.G.1.d., supra). Moreover, warranty disclaimers that limit damages for personal injury caused by a breach of warranty on consumer goods are prima facie unconscionable.5) Timing - Disclaimers and Limitations in the BoxTo be effective, a disclaimer of warranty or limitation on remedies must be agreed to during the bargaining process. Thus, although a few courts hold otherwise, most hold that a warranty disclaimer or limitation on remedy included inside the packaging of goods is not effective against the buyer. a) Compare - "Clickwrap"Computer software often comes with terms that appear on the user's computer screen during the installation process, and the purchaser must click to agree to the terms before installing. Such limitations and disclaimers typically are upheld on the rationale that the purchaser can return the software if he disagrees with the conditions. 6) Unconscionability and Warranty DisclaimersSome courts will, in addition to determining whether disclaimers have met the formal requirements discussed above, test warranty disclaimers by the unconscionability standards. (See IV.G., supra.) CHART f. Damages for Breach of Warranty 1) In General - Difference Between Goods Tendered and as WarrantedGenerally, the measure of damages for breach of any warranty is the difference between the value of the goods accepted and the value of the goods as warranted, measured at the time and place of acceptance. If there are special circumstances, damages may be measured differently to account for those circumstances.2) Breach of Warranty of TitleIf the warranty of title is breached, the goods are reclaimed by the true owner or lienholder, thus dispossessing the buyer. The buyer may then rescind the contract, revoke acceptance of the goods, or sue for damages. The value of the goods accepted is deemed to be nothing); so the damages are the value of the goods as warranted. Often, but not always, that is the same as the purchase price. a) Special Circumstances - Appreciation and DepreciationIf there are special circumstances, the value of the goods is measured at the time of the dispossession rather than at the time of acceptance. A great appreciation (e.g., art) or depreciation (e.g., car) in the value of the goods from the time of delivery until dispossession is usually considered a special circumstance. g. To Whom Do Warranties Extend?UCC section 2-318 provides alternative provisions for determining to whom warranty liability extends. Most states have adopted the narrowest provision, Alternative A, which provides that the seller's warranty liability extends to any natural person who is in the family or household of the buyer or who is a guest in the buyer's home if it is reasonable to expect that the person may use, consume, or be affected by the goods and that person suffers personal injury because of a breach of warranty. E. MODIFICATION OF CONTRACT TERMS1. ConsiderationUnder general contract law, a contract cannot be modified unless the modification is supported by new consideration. The modern view, however, permits modification without consideration if: (i) the modification is due to circumstances that were unanticipated by the parties when the contract was made and (ii) it is fair and equitable. The UCC is even more liberal - good faith promises of new and different terms by the parties to a sales contract are valid without consideration.EXAM TIPFor MBE purposes, the examiners have indicated that they have adopted the modern view. However, on any non-UCC essay question that involves modification, you should discuss the traditional view and any relevant state exceptions, including the modern Restatement view where it is relevant.2. WritingA written contract can be modified orally. However, the modification must be in writing if the contract as modified falls within the Statute of Frauds. Thus, for a sale of goods contract, if the contract as modified is for $500 or more, it must be evidenced by a writing); if the contract as modified is for less than $500, no writing is necessary. a. Common Law - Provisions Prohibiting Oral Modification Not EffectiveThe common law rule is that even if a written contract expressly provides that it may be modified only by a writing, the parties can orally modify the contract.b. UCC - No-Modification Clauses EffectiveUnder the UCC, if a contract explicitly provides that it may not be modified or rescinded except by a signed writing, that provision is given effect. If the contract is between a merchant and nonmerchant, however, this provision requires the nonmerchant's separate signature.c. WaiverIf the parties attempt to orally modify a contract that requires written modification (either because of a contract clause or the Statute of Frauds), it is technically ineffective as a modification, but it can operate as a waiver. Such a waiver will be found whenever the other party has changed position in reliance on the oral modification. However, a party who makes a waiver affecting an executory (not yet performed) portion of the contract may retract the waiver if she notifies the other party that strict performance of the waived terms is required. The waiver may not be retracted if the other party detrimentally relied on it. 3. Parol Evidence Rule Does Not ApplyAs noted above, parol evidence is admissible to show subsequent oral modifications of a written contract.

o be enforceable under the Statute of Frauds, which of the following must be in a writing signed by the parties sought to be bound? AAll leases of real estate for more than one year BAll leases of real estate COnly agreements for the sale of real estate, not leases DAll leases of real estate for more than six months

A Under the Statute of Frauds, a promise creating an interest in land must be evidenced by a writing. This includes not only agreements for the sale of real property, but also other agreements pertaining to land, such as leases for more than one year. It is not necessary that all leases, such as a six-month lease, be in writing. Only those leases of real estate for more than one year are covered by the Statute.

Under the confirmatory memo rule, in contracts between merchants, if one party, within a reasonable time after an oral agreement has been made, sends to the other party a written confirmation of the understanding that is sufficient under the Statute of Frauds to bind the sender, it will also bind the recipient if: AThe recipient has reason to know of the confirmation's contents and does not object to it in writing within 10 days of receipt BThe recipient has knowledge of the confirmation's contents and confirms the contents in a signed writing within 10 days of receipt CThe recipient knows or has reason to know of the confirmation's contents and admits orally to the substance of the memo in court testimony

A Under the confirmatory memo rule, in contracts between merchants, if one party, within a reasonable time after an oral agreement has been made, sends to the other party a written confirmation of the understanding that is sufficient under the Statute of Frauds to bind the sender, it will also bind the recipient if the recipient has reason to know of the confirmation's contents and does not object to it in writing within 10 days of receipt. If the recipient were to confirm the contents in a signed writing, the confirmatory memo rule would no longer be necessary because that writing could be used to satisfy the Statute of Frauds. Similarly, if the recipient admits orally to the substance of the memo in court testimony, the contract is enforceable without use of the confirmatory memo rule. When a party admits in pleadings, testimony, or otherwise in court that an oral contract was made, the contract is enforceable without a writing, but only up to the quantity of goods admitted.

Under the warranty of _____________ , goods are warranted to be fit for the ordinary purpose for which such goods are used. AMerchantability BFitness for a particular purpose CAgainst infringement

A Under the implied warranty of merchantability, the goods are warranted to be at least fit for the ordinary purposes for which such goods are used. The warranty against infringement is provided by a merchant seller and warrants that the goods are delivered free of any patent, trademark, copyright, or similar claims. The implied warranty of fitness for a particular purpose arises when a seller has reason to know the particular purpose for which the goods are to be used and that the buyer was relying on the seller's skill and judgment to select suitable goods when the buyer bought the goods. This warranty is for a specific use intended by the buyer, whereas the implied warranty of merchantability warrants fitness for ordinary purposes. A. INTRODUCTIONOnce you have determined that a contract exists, the next thing you must do is determine what its terms are.B. GENERAL RULES OF CONTRACT CONSTRUCTIONThere are a number of general rules of construction applied by the courts when interpreting contracts. The following are among the more frequently invoked: 1. Contracts will be construed as a "whole"); specific clauses will be subordinated to the contract's general intent);2. The courts will construe words according to their "ordinary" meaning unless it is clearly shown that they were meant to be used in a technical sense);3. If provisions appear to be inconsistent, written or typed provisions will prevail over printed provisions);4. It is important to note that the courts generally will try to reach a determination that a contract is valid and enforceable);5. Ambiguities in a contract are construed against the party preparing the contract, absent evidence of the intention of the parties);6. The parties' course of dealing (i.e., the sequence of conduct concerning previous transactions between the parties to a particular transaction that may be regarded as establishing a common basis of their understanding));7. A usage of trade (i.e., a practice or method of dealing, regularly observed in a particular business setting so as to justify an expectation that it will be followed in the transaction in question));8. The parties' course of performance (i.e., if a contract involves repeated occasions for performance by either party and the other party has the opportunity to object to such performance, any course of performance accepted or acquiesced to is relevant in determining the meaning of the contract)); and9. When rules conflict: (i) express terms are given greater weight than course of performance, course of dealing, and usage of trade); (ii) course of performance is given greater weight than course of dealing or usage of trade); and (iii) course of dealing is given greater weight than usage of trade. C. PAROL EVIDENCE RULE - SUPPLEMENTING, EXPLAINING, OR CONTRADICTING TERMSWhen the parties to a contract express their agreement in a writing with the intent that it embody the final expression of their bargain (i.e., the writing is an "integration"). Any other expressions - written or oral - made prior to the writing, as well as any oral expressions contemporaneous with the writing, are inadmissible to vary the terms of the writing. 1. Is the Writing an "Integration"?There are two components: (i) whether the writing was intended as the final expression of the agreement); and (ii) whether the integration was intended to be complete or partial. Evidence is admissible to show the parties' intent. a. Partial Integration - Additional Terms PermittedIf an integration is complete, the writing cannot be contradicted or supplemented. If, however, the integration is partial, the writing may not be contradicted but may be supplemented by proving consistent additional terms. The UCC presumes all writings are partial integrations. b. Effect of Merger ClauseA merger clause recites that the agreement is the complete agreement between the parties. The presence of a merger clause is usually determinative in large commercial contracts. For most contracts, however, the modern trend is to consider it as one factor in determining integration. EXAM TIPA memo prepared by one party and not shown to the other can never be an integration because the parties could not have intended it to be the final complete expression of their agreement when one party has not even seen it. The writing is merely evidence of the agreement. Note that a confirmatory memo may be a partial integration under the UCC because it was sent to the other party and that party was aware of its contents. 2. Evidence Outside Scope of RuleBecause the rule prohibits admissibility only of extrinsic evidence that seeks to vary, contradict, or add to an "integration," other forms of extrinsic evidence may be admitted if they will not bring about this result, i.e., they will fall outside the scope of the parol evidence rule. a. Validity IssuesA party to a written contract can attack the agreement's validity. The party acknowledges (concedes) that the writing reflects the agreement but asserts, most frequently, that the agreement never came into being because of any of the following: 1) Formation DefectsFormation defects (e.g., fraud, duress, mistake, and illegality) may be shown by extrinsic evidence.2) Conditions Precedent to EffectivenessIf a party asserts that there was an oral agreement that the written contract would not become effective until a condition occurred, all evidence of the understanding may be offered and received. b. Collateral Agreements and Naturally Omitted TermsParol evidence is often said to be admissible if the alleged parol agreement is collateral to the written obligation (i.e., related to the subject matter but not part of the primary promise) and does not conflict with it. The Restatements of Contracts include a similar concept with a more definitive approach: the naturally omitted terms doctrine. The doctrine allows evidence of terms that would naturally be omitted from the written agreement. A term would naturally be omitted if: (i) it does not conflict with the written integration); and (ii) it concerns a subject that similarly situated parties would not ordinarily be expected to include in the written instrument.c. InterpretationIf there is uncertainty or ambiguity in the written agreement's terms or a dispute as to the meaning of those terms, parol evidence can be received to aid the fact-finder in reaching a correct interpretation of the agreement. However, if the meaning of the agreement is plain, parol evidence is inadmissible.d. Showing of "True Consideration"The parol evidence rule will not bar extrinsic evidence showing the "true consideration" paid (e.g., evidence that the consideration stated in the contract was never paid).e. ReformationIf a party to a written agreement alleges facts (e.g., mistake) entitling him to reformation of the agreement, the parol evidence rule is inapplicable.f. Subsequent ModificationsParol evidence can be offered to show subsequent modifications of a written contract.g. Additional Terms Under Article 2Article 2 generally follows the rules discussed above, providing that a party cannot contradict a written contract but may add consistent additional terms unless: (i) there is a merger clause, or (ii) the courts find from all of the circumstances that the writing was intended as a complete and exclusive statement of the terms of the agreement. Article 2 also provides that a written contract's terms may be explained or supplemented by evidence of course of performance, course of dealing, and usage of trade-regardless of whether or not the writing appears to be ambiguous. D. ARTICLE 2 PROVISIONS ON INTERPRETING CONTRACTS 1. Supplemental ("Gap Filler") TermsRecall that the key to forming a contract for the sale of goods is the quantity term (see II.B.2.b.1)b), supra). If other terms are missing from the agreement, Article 2 has gap filler provisions to fill in the missing term(s). a. PriceIf: (i) nothing has been said as to price); (ii) the price is left open to be agreed upon by the parties and they fail to agree); or (iii) the price is to be fixed in terms of some standard that is set by a third person or agency and it is not set, then the price is a reasonable price at the time for delivery.b. Place of DeliveryIf the place of delivery is not specified, the place usually is the seller's place of business, if he has one); otherwise, it is the seller's home.c. Time for Shipment or DeliveryIf the time for shipment or delivery is not specified, shipment/delivery is due in a reasonable time.d. Time for PaymentIf the time for payment is not specified, payment is due at the time and place at which the buyer is to receive the goods.e. AssortmentIf a contract provides that an assortment of goods is to be delivered (e.g., blouses in various colors and sizes) and does not specify which party is to choose, the assortment is at the buyer's option. If the party who has the right to specify the assortment does not do so seasonably, the other party is excused from any resulting delay and may either proceed in any reasonable manner (e.g., choose a reasonable assortment) or treat the failure as a breach. 2. Delivery Terms and Risk of LossAll contracts for the sale of goods require delivery of the goods. A contract's delivery terms are important because they determine when risk of loss passes from the seller to the buyer if the goods are damaged or destroyed. a. Noncarrier CaseA noncarrier case is a sale in which it appears that the parties did not intend that the goods would be moved by a common carrier (e.g., when you buy groceries). In such a case, if the seller is a merchant, risk of loss passes to the buyer only when she takes physical possession of the goods. If the seller is not a merchant, risk of loss passes to the buyer upon tender of delivery.b. Carrier CaseA carrier case is a sale in which it appears that the parties intended the goods to be moved by a carrier (e.g., when you order a book from an Internet website). There are two types of carrier cases: shipment contracts and destination contracts. 1) Shipment ContractIf the contract authorizes or requires the seller to ship the goods by carrier but does not require him to deliver them at a particular destination, it is a shipment contract and risk of loss passes to the buyer when the goods are delivered to the carrier. In the absence of a contrary agreement, Article 2 presumes a contract is a shipment contract. a) Seller's Duties Under Shipment ContractIn a shipment contract, the seller must: (i) make a reasonable contract with the carrier on behalf of the buyer); (ii) deliver the goods to the carrier); (iii) promptly notify the buyer of the shipment); and (iv) provide the buyer with any documents needed to take possession of the goods. 2) Destination ContractsIf the contract requires the seller to deliver the goods at a particular destination, the risk of loss passes to the buyer when the goods are tendered to the buyer at the destination.3) Common Delivery Terms a) F.O.B.F.O.B. stands for "free on board." The letters F.O.B. are always followed by a location, and the risk of loss passes to the buyer at the named location. The seller bears the risk and expense of getting the goods to the named location. These contracts can be either shipment contracts or destination contracts, depending on the location named.b) F.A.S.F.A.S. stands for "free alongside." The term is generally used only when goods are to be shipped by boat. Risk of loss passes to the buyer once the goods are delivered to the dock. EXAM TIPAll contracts for goods require an address for delivery. Merely indicating an address for shipment does not make a contract a destination contract. A contract that does not contain an F.O.B. term or any other term explicitly allocating the risk of loss is a shipment contract. CHART c. Effect of Breach on Risk of Loss 1) Defective GoodsIf the buyer has a right to reject the goods, the risk of loss does not pass to the buyer until the defects are cured or she accepts the goods in spite of their defects. Note that a buyer generally has the right to reject for any defect. (See VII.C., infra.)2) Revocation of AcceptanceIf the buyer rightfully revokes acceptance, the risk of loss is treated as having rested on the seller from the beginning to the extent of any deficiency in the buyer's insurance coverage. EXAM TIPBecause of the above rules, if a seller ships nonconforming goods, it eliminates the importance of determining whether a contract is a shipment or destination contract. If the goods are nonconforming, the risk of loss remains on the seller.d. Risk in Sale or Return and Sale on Approval Contracts 1) Sale or ReturnFor the purpose of determining the risk of loss, a sale or return contract (e.g., the buyer takes goods for resale but may return them if she is unable to resell them) is treated as an ordinary sale and the above rules apply. If the goods are returned to the seller, the risk remains on the buyer while the goods are in transit.2) Sale on ApprovalIn a sale on approval (i.e., the buyer takes goods for use but may return them even if they conform to the contract), the risk of loss does not pass to the buyer until she accepts. CHART e. Goods Destroyed Before Risk of Loss PassesIf goods that were identified when the contract was made are destroyed (i) without fault by either party and (ii) before the risk of loss passes to the buyer, the contract is avoided (i.e., the seller's performance is excused). If the goods were not identified until after the contract was made, the seller in this situation would have to prove impracticability (VI.E.4.b., infra) to be discharged. 3. Insurable Interest and IdentificationAs noted above, a buyer often bears the risk of loss before receiving the goods purchased. In order to aid buyers in this situation (and a few others), Article 2 gives buyers a special property interest in goods as soon as they are identified as the ones that will be used to satisfy the contract (e.g., as soon as the seller sets them aside for the buyer). This special property interest is insurable.4. Bilateral Contracts Formed by PerformanceRecall that a contract may be formed by the parties' performance where the mirror image rule is not satisfied and under certain circumstances under Article 2's "battle of the forms" provision. (See II.D.5.b., supra.) In such cases, under Article 2, the contract includes all of the terms on which the writings of both parties agree. Any necessary missing terms are filled in by the supplemental terms provided for in Article 2. a. Compare - Common Law Last Shot RuleThe rule is different in common law contracts. At common law, the contract includes the terms of the last communication sent to the party who performed. 5. WarrantiesContracts for the sale of goods automatically include a warranty of title (in most cases). They also may include certain implied warranties and express warranties. a. Warranty of Title and Against Infringement 1) Warranty of TitleAny seller of goods warrants that the title transferred is good, that the transfer is rightful, and that there are no liens or encumbrances against the title of which the buyer is unaware at the time of contracting. This warranty arises automatically and need not be mentioned in the contract.2) Warranty Against InfringementA merchant seller regularly dealing in goods of the kind sold also automatically warrants that the goods are delivered free of any patent, trademark, copyright, or similar claims. But a buyer who furnishes specifications for the goods to the seller must hold the seller harmless against such claims. b. Implied Warranty of MerchantabilityImplied in every contract for sale by a merchant who deals in goods of the kind sold, there is a warranty that the goods are merchantable. To be merchantable, goods must at least be "fit for the ordinary purposes for which such goods are used." 1) Seller's Knowledge of Defect Not RelevantAs in all implied warranty cases, it makes no difference that the seller himself did not know of the defect or that he could not have discovered it. Implied warranties are not based on negligence but rather on absolute liability that is imposed on certain sellers. c. Implied Warranty of Fitness for a Particular PurposeA warranty will also be implied in a contract for the sale of goods whenever (i) any seller, merchant or not, has reason to know the particular purpose for which the goods are to be used and that the buyer is relying on the seller's skill and judgment to select suitable goods); and (ii) the buyer in fact relies on the seller's skill or judgment.d. Express WarrantiesAny affirmation of fact or promise made by the seller to the buyer, any description of the goods, and any sample or model creates an express warranty if the statement, description, sample, or model is part of the basis of the bargain. For the statement, description, sample, or model to be a part of the basis of the bargain, it need only come at such a time that the buyer could have relied on it when she entered into the contract. The buyer does not need to prove that she actually did rely, although the seller may negate the warranty by proving that the buyer as a matter of fact did not rely. It is not necessary that the seller intended the affirmation of fact, description, model, or sample to create a warranty. 1) Distinguish - Statements of Value or OpinionA statement relating merely to the value of the goods, or a statement purporting to be only the seller's opinion or commendation of the goods, does not create an express warranty. e. Disclaimer of Warranties 1) Warranty of TitleThe title warranty can be disclaimed or modified only by specific language or by circumstances that give the buyer notice that the seller does not claim title or that he is selling only such rights as he or a third party may have (e.g., a sheriff's sale).2) Implied WarrantiesThe implied warranties of merchantability and fitness for a particular purpose can be disclaimed by either specific disclaimers or general methods of disclaimer. a) Specific Disclaimers (1) Disclaimer of Warranty of MerchantabilityThe warranty of merchantability can be specifically disclaimed or modified only by mentioning merchantability. If the sales contract is in writing, the disclaimer must be conspicuous.(2) Disclaimer of Warranty of Fitness for a Particular PurposeThe warranty of fitness for a particular purpose can be specifically disclaimed only by a conspicuous writing. A written disclaimer, according to the statute, is sufficient if it says, for example, "[t]here are no warranties which extend beyond the description on the face hereof."(3) "Conspicuous" DefinedA term is conspicuous when it is "so written, displayed, or presented that a reasonable person against whom it is to operate ought to have noticed it." Language in the body of a writing is conspicuous if: (i) it is in larger type than surrounding text); (ii) it is in a contrasting type, font, or color); or (iii) it is set off from the text by marks that call attention to it. The court, not the jury, decides any fact question as to conspicuousness. b) Other Methods of Disclaiming Implied WarrantiesThe UCC also provides several more general methods for disclaiming implied warranties. (1) By "As Is" or Similar LanguageUnless the circumstances indicate otherwise, the implied warranties of merchantability and fitness can be disclaimed by expressions such as "as is," "with all faults," or other expressions that in common understanding call the buyer's attention to the fact that there are no implied warranties. Although this type of disclaimer does not have to be conspicuous, a hidden or fine-print disclaimer of this type is not effective.(2) By Examination or Refusal to ExamineIf the buyer, before entering into the contract, has examined the goods or a sample or model as fully as she desires or has refused to examine, there is no warranty as to defects that a reasonable examination would have revealed to her.(3) By Course of Dealing, Etc.Implied warranties may also be disclaimed by the course of dealing, course of performance, or usage of trade. EXAM TIPIt may seem odd that there are specific disclaimer methods with detailed requirements, and more general disclaimer methods requiring little formality. In actual practice, it is better to use the specific disclaimers because general disclaimers may be limited by the circumstances. However, on the MBE, an "as is" or "with all faults" disclaimer will generally be as effective as a specific disclaimer. 3) Express WarrantiesThe UCC provides that words or conduct relevant to the creation of express warranties and words or conduct tending to negate such warranties shall wherever possible be construed as consistent with each other, but "negation or limitation is inoperative to the extent that such construction is unreasonable." In other words, once an express warranty is made, it is very difficult to disclaim.4) Limitations on DamagesParties may include in their contract a clause limiting the damages available in the case of breach of warranty (e.g., "remedy for breach of warranty is limited to repair or replacement of the defective goods"). However, such a limitation will not be upheld if it is unconscionable (e.g., causes the remedy to fail of its essential purpose, limits personal injury damages for consumer goods); see IV.G.1.d., supra). Moreover, warranty disclaimers that limit damages for personal injury caused by a breach of warranty on consumer goods are prima facie unconscionable.5) Timing - Disclaimers and Limitations in the BoxTo be effective, a disclaimer of warranty or limitation on remedies must be agreed to during the bargaining process. Thus, although a few courts hold otherwise, most hold that a warranty disclaimer or limitation on remedy included inside the packaging of goods is not effective against the buyer. a) Compare - "Clickwrap"Computer software often comes with terms that appear on the user's computer screen during the installation process, and the purchaser must click to agree to the terms before installing. Such limitations and disclaimers typically are upheld on the rationale that the purchaser can return the software if he disagrees with the conditions. 6) Unconscionability and Warranty DisclaimersSome courts will, in addition to determining whether disclaimers have met the formal requirements discussed above, test warranty disclaimers by the unconscionability standards. (See IV.G., supra.) CHART f. Damages for Breach of Warranty 1) In General - Difference Between Goods Tendered and as WarrantedGenerally, the measure of damages for breach of any warranty is the difference between the value of the goods accepted and the value of the goods as warranted, measured at the time and place of acceptance. If there are special circumstances, damages may be measured differently to account for those circumstances.2) Breach of Warranty of TitleIf the warranty of title is breached, the goods are reclaimed by the true owner or lienholder, thus dispossessing the buyer. The buyer may then rescind the contract, revoke acceptance of the goods, or sue for damages. The value of the goods accepted is deemed to be nothing); so the damages are the value of the goods as warranted. Often, but not always, that is the same as the purchase price. a) Special Circumstances - Appreciation and DepreciationIf there are special circumstances, the value of the goods is measured at the time of the dispossession rather than at the time of acceptance. A great appreciation (e.g., art) or depreciation (e.g., car) in the value of the goods from the time of delivery until dispossession is usually considered a special circumstance. g. To Whom Do Warranties Extend?UCC section 2-318 provides alternative provisions for determining to whom warranty liability extends. Most states have adopted the narrowest provision, Alternative A, which provides that the seller's warranty liability extends to any natural person who is in the family or household of the buyer or who is a guest in the buyer's home if it is reasonable to expect that the person may use, consume, or be affected by the goods and that person suffers personal injury because of a breach of warranty. E. MODIFICATION OF CONTRACT TERMS1. ConsiderationUnder general contract law, a contract cannot be modified unless the modification is supported by new consideration. The modern view, however, permits modification without consideration if: (i) the modification is due to circumstances that were unanticipated by the parties when the contract was made and (ii) it is fair and equitable. The UCC is even more liberal - good faith promises of new and different terms by the parties to a sales contract are valid without consideration.EXAM TIPFor MBE purposes, the examiners have indicated that they have adopted the modern view. However, on any non-UCC essay question that involves modification, you should discuss the traditional view and any relevant state exceptions, including the modern Restatement view where it is relevant.2. WritingA written contract can be modified orally. However, the modification must be in writing if the contract as modified falls within the Statute of Frauds. Thus, for a sale of goods contract, if the contract as modified is for $500 or more, it must be evidenced by a writing); if the contract as modified is for less than $500, no writing is necessary. a. Common Law - Provisions Prohibiting Oral Modification Not EffectiveThe common law rule is that even if a written contract expressly provides that it may be modified only by a writing, the parties can orally modify the contract.b. UCC - No-Modification Clauses EffectiveUnder the UCC, if a contract explicitly provides that it may not be modified or rescinded except by a signed writing, that provision is given effect. If the contract is between a merchant and nonmerchant, however, this provision requires the nonmerchant's separate signature.c. WaiverIf the parties attempt to orally modify a contract that requires written modification (either because of a contract clause or the Statute of Frauds), it is technically ineffective as a modification, but it can operate as a waiver. Such a waiver will be found whenever the other party has changed position in reliance on the oral modification. However, a party who makes a waiver affecting an executory (not yet performed) portion of the contract may retract the waiver if she notifies the other party that strict performance of the waived terms is required. The waiver may not be retracted if the other party detrimentally relied on it. 3. Parol Evidence Rule Does Not ApplyAs noted above, parol evidence is admissible to show subsequent oral modifications of a written contract.

A bargained-for change in legal position between the parties is commonly known as __________. Avaluable consideration Ban executory bilateral contract Cpromissory estoppel Ddetrimental reliance

A Valuable consideration can be defined as a bargained-for change in legal position between the parties. Valuable consideration is only one element of a properly formed executory bilateral contract. The substitute doctrines of detrimental reliance and promissory estoppel might come into play when valulable consideration is not present, but when the facts indicate that to prevent injustice the promisor should be estopped from not performing.

When both parties entering into a contract are mistaken about an existing fact relating to the agreement, __________. Athe contract is voidable if the mistake concerns a basic assumption on which the contract is made Bthe contract is voidable if the mistake concerns the value of the subject matter of the contract Cthe contract is void if the mistake concerns a basic assumption on which the contract is made Dthe contract is void if the mistake concerns the value of the subject matter of the contract

A When both parties entering into a contract are mistaken about an existing fact relating to the agreement, the contract is voidable by the adversely affected party if the mistake concerns a basic assumption on which the contract is made. A mistake concerning a basic assumption on which the contract is made makes the contract voidable, not void. If the parties to a contract make a mistaken assumption as to the value of the subject matter, that mistake generally will not be remedied. Thus, in that case, the contract is neither voidable nor void.

Which of the following would serve as sufficient consideration for a promise by a creditor to discharge an existing debt? AAn alternative method of payment BUnforeseen difficulty in performance by the debtor CPartial payment of the debt DAcknowledgement of the existence of the debt

A When the proposed consideration is in any way new or different (e.g., an alternative method of payment), there is usually sufficient consideration to change a preexisting duty, such as discharging an existing debt. Mere acknowledgement of a preexisting duty is not sufficient consideration to change the preexisting duty.Partial payment of the amount due on an existing debt is not sufficient consideration for a promise by the creditor to discharge the debt. Neither a legal detriment nor benefit is present. Under the majority view, mere unforeseen difficulty in performance is not a substitute for consideration. Although the modern view permits modification without consideration if the modification is fair and equitable in view of circumstances not anticipated when the contract was made, it would not apply to payment of an existing debt. That exception to the consideration requirement applies only if the contract has not been fully performed on either side, and an existing debt suggests that the creditor has already performed. Also, as with impracticability, difficulty in paying money would be unlikely to be considered the type of unforeseen circumstance this view is intended to addre

Which of the following will always render a contract voidable? AA mutual mistake as to the value of the subject matter of the contract BA unilateral mistake as to a basic assumption on which the contract is made CA unilateral mistake as to the value of the subject matter of the contract DA mutual mistake as to a basic assumption on which the contract is made

D A mutual mistake as to a basic assumption on which the contract is made will render a contract voidable by the adversely affected party. If only one of the parties is mistaken about facts relating to the agreement (i.e., a unilateral mistake), that mistake will not prevent formation of a contract unless the nonmistaken party knew or had reason to know of the mistake made by the other party. If the parties to a contract make a mistaken assumption as to the value of the subject matter of the contract, that mistake generally will not be remedied, whether such mistake is mutual or unilateral.

A child wishes to buy an electric guitar from a music store on credit. Her father promises the music store that he will make the payments on the guitar if the child defaults. The father's promise is __________. AAn illusory promise BAn option contract CA voidable contract DA suretyship promise

D The father's promise is a suretyship promise because it is a promise to pay the debt of another. Although the child is a minor and her contract with the music store may be voidable, the father is under no such disability and, thus, his suretyship promise is not voidable. The father's promise is not illusory. An illusory promise is one in which the promisor is not actually bound to perform. Here, the father is bound to make the payments if the child defaults. This is not an option contract. An option contract is a distinct contract in which the offeree gives consideration for a promise by the offeror not to revoke the offer.

Which of the following is an element of the implied warranty of fitness for a particular purpose? AThe goods must pass without objection in the trade under the contract description BThe seller is a merchant CThe goods must conform to any promises or affirmations of fact made on the label DThe buyer relied on the seller's skill and judgment

D The warranty of fitness for a particular purpose is implied when:(i) any seller (merchant or not) has reason to know the particular purpose for which the goods are to be used and that the buyer was relying on the seller's skill and judgment to select suitable goods; and (ii) the buyer in fact relied on the seller's skill or judgment. The warranty that the goods will pass without objection in the trade under the contract description and the warranty that the goods conform to any promises or affirmations of fact made on the label are both elements of the implied warranty of merchantability, not fitness for a particular purpose.

Under the common law of contracts, modification of a written contract generally requires __________. Aa waiver if the contract provides that all modifications must be in writing Bconsideration Ca signed writing Dgood cause

B Contrary to the U.C.C. position, the common law of contracts generally requires that a modification be supported by new consideration. A signed writing is not required. A written contract may be modified orally. The common law rule is that even if a written contract provides that all modifications must be in writing, the parties can orally modify the contract. Good cause is not required; a contract may be modified for any reason at all.

In determining whether an agreement qualifies as a legally enforceable contract, courts of law normally will not inquire into the __________ of consideration. Amutuality Badequacy Cpresence Dtiming

B Courts of law normally will not inquire into the adequacy of consideration, such as when one party wishes to contract to sell an item of high market value for a relatively low price. Consideration on both sides of the bargain will make an executory bilateral contract fully enforceable from the moment of formation. Thus, the presence of consideration is a necessary element of contract formation. If something was already given or performed before the promise was made, it will not satisfy the "bargain" requirement. Therefore the timing of consideration is an issue courts will consider. Consideration must exist on both sides of the contract, or in other words courts consider the issue of mutuality.

Which of the following is sufficient to establish bargained-for consideration? AA benefit that is not intended to induce a detriment BA benefit that provides peace of mind CA detriment involving a prior legal obligation

B For valid consideration, the benefit need not be economic. A benefit of peace of mind or the gratification of influencing the mind of another is sufficient to establish bargained-for consideration, provided that the promisee is not already legally obligated to perform the requested act. Prior legal obligations, sometimes called "past consideration," generally are not sufficient consideration. If something was already given or performed before the promise was made, it was not given in "exchange" for the promise when made. To constitute bargained-for consideration, the detriment must be the price of the exchange. If the promisor's motive was to induce the detriment, it is consideration. However, if the motive was a condition of a promise for a gift (e.g., "come to my house, and I will give you my old stereo"), there is no consideration.

At common law, the Statute of Frauds requires _____________ signed by ____________. AA formal contract; the party to be held liable BA writing or writings reflecting the material terms of the contract; the party to be held liable CA writing or writings reflecting the material terms of the contract; both parties DA formal contract; both parties

B To satisfy the Statute of Frauds, there must be one or more writings that reflect the material terms of the contract signed by the person sought to be held liable on the contract. The Statute does not require both parties to sign, only the party to be charged. The Statute of Frauds does not require a formal written contract or the signature of both parties. For example, a letter, receipt, or a check containing the material terms (e.g., quantity for sale of goods) and signed by the party to be charged satisfies the Statute of Frauds.

Generally speaking, the promise to perform an existing legal duty is __________. APast consideration BNot consideration CValuable consideration DSufficient consideration

B Traditionally the promise to perform, or the performance of, an existing legal duty is not consideration. A promise to perform an existing legal duty is not valuable consideration, unless an exception to the preexisting legal duty rule applies, e.g., new or different consideration is promised, or a minor's ratification of a voidable contract upon reaching the age of majority. Past consideration, which is also not sufficient consideration, is based on something already given or performed, not a promise to perform based on a preexisting legal duty.

Under U.C.C. Article 2, what standard is applied to allow the modification of a sales contract without additional consideration? ADetrimental reliance BGood faith CHonest dispute

B Under Article 2, all contract modifications sought in good faith are binding without consideration. (In contrast, at common law, a contract modification generally is unenforceable unless it is supported by new consideration.) A compromise based on an honest dispute as to the legal duty owed is adequate new consideration for a contract, not the standard for modification of a sales contract without consideration. Detrimental reliance is a means of enforcing a promise without any consideration under certain circumstances when no actual contract has been formed. It is not the standard for allowing the modification of a sales contract without additional consideration under Article 2.

Which of the following contracts must be evidenced in writing? AA contract between business partners to buy and sell real estate and divide the profits BA mortgage contract CA contract to build a building DA six-month lease of a parcel of land

B Under the Statute of Frauds, a promise creating an interest in land must be evidenced by a writing. This includes not only agreements for the sale of real property, but also other agreements pertaining to land, such as a mortgage contract. Some contracts may have an end result involving an interest in land, but they still do not come within the Statute. For example, a contract to build a building or a contract to buy and sell real estate and divide the profits do not come within the Statute. A lease of a parcel of land for more than one year is also covered by the Statute, but a six-month lease is not.

Which of the following is the best definition for the term "consideration"? AAn existing legal duty BA bargained-for change in legal position between the parties CAny action that is reasonably expected to induce action or forbearance

B Valuable consideration can be defined as a bargained-for change in legal position between the parties. Under the theory of promissory estoppel, a promise can be enforceable without consideration if necessary to prevent injustice, if the promisor should reasonably expect to induce action or forbearance, of a definite and substantial character, and such action or forbearance is in fact induced. Traditionally the promise to perform, or the performance of, an existing legal duty is not consideration. This is also known as the preexisting legal duty rule.

Which of the following statements could give rise to an express warranty? A"Ford Fusions have the best resale value" B"Your car will look just like the one in the showroom" C"In my opinion, this is the best car with the most fuel-efficient engine available today"

B "Your car will look just like the one in the showroom" is a description that could give rise to an express warranty if made at a time that the buyer could have relied on it when entering into the contract. It is a statement of fact about the good. Statements relating merely to the value of the goods, such as "Ford Fusions have the best resale value," and statements purporting to be only the seller's opinion, such as "In my opinion, this is the best car with the most fuel-efficient engine available today," do not create express warranties.

A promise to choose one of several alternative means of performance is illusory (lacks consideration) if __________. Athe power to choose the means of performance rests with the promisee Bthe promisor retains the power to select an alternative without legal detriment Cevery alternative involves some legal detriment to the promisor Dsome of the alternatives involve no legal detriment, regardless of who has the power to choose

B A promise to choose one of several alternative means of performance is illusory if the promisor retains the power to select an alternative without legal detriment. Ordinarily, a promise to choose one of several alternative means of performance is illusory unless every alternative involves some legal detriment to the promisor. However, if the power to choose rests with the promisee or some third party not under the control of the promisor, the promise is enforceable even though some alternatives involve no legal detriment, as long as at least one alternative involves some legal detriment.

A promise __________ lacks consideration. Awithout an economic benefit Bto make a gift Cbased on peace of mind

B A promise to make a gift lacks consideration because there is no bargained-for exchange. A promise without an economic benefit may be proper consideration. A promise based on peace of mind or the gratification of influencing the mind of another may be sufficient to establish bargained-for consideration, provided that the promisee is not already legally obligated to perform the requested ac

A suretyship contract is supported by proper consideration: AOnly if the surety is compensated BIf the surety is compensated or makes the suretyship promise before (or at the same time as) the creditor performs or promises to perform CIf the surety is compensated or makes the suretyship promise after the creditor performs or promises to perform

B A suretyship contract must be supported by consideration. Compensation will serve as proper consideration for a surety's promise. In addition, if a gratuitous surety makes his promise to pay before (or at the same time as) the creditor performs or promises to perform, the creditor's performance or promise will serve as proper consideration for the surety's promise, because the creditor has incurred a detriment in exchange for the surety's promise. In contrast, if a gratuitous surety does not make his promise until after the creditor has performed or made an absolute promise to perform, there is no consideration to support the surety's promise because of the preexisting legal duty rule.

Which of the following is a true statement about express warranties? AThe buyer must have relied on the statement or affirmation when the buyer entered into the contract BA sample or model can create an express warranty CThe seller must intend that the statement, affirmation of fact, or description create a warranty DThe statement or affirmation may relate to the value of the goods

B Any affirmation of fact or promise made by the seller to the buyer, any description of the goods, and any sample or model creates an express warranty if the statement, description, sample, or model is part of the basis of the bargain. The buyer does not have to prove that she actually relied on the statement or affirmation, only that it came at such a time that she could have relied on it when she entered into the contract. It is not necessary that the seller intended the affirmation of fact, description, model, or sample to create a warranty. A statement relating to the value of the goods does not create an express warranty.

Conditional promises are enforceable even if __________, but not if __________. AThe condition is entirely within the promisor's control; the contingency is extremely remote BThe contingency is extremely remote; the condition is entirely within the promisor's control CThe condition is entirely within the promisee's control; the contingency is extremely remote DThe contingency is extremely remote; the condition is entirely within the promisee's control

B Conditional promises are enforceable, no matter how remote the contingency, unless the "condition" is entirely within the promisor's control. Such a promise will be deemed illusory. An illusory promise is one in which the promisor is not actually bound to perform. The promisor could simply choose to assert his control over the condition so that he suffers no legal detriment. Consideration fails in such an instance because the agreement lacks mutuality. Mutuality requires that consideration exists on both sides of the contract. If the condition is entirely within the promisee's (or some third party's) control, the promise is enforceable as long as it involves a possibility of legal detriment, no matter how remote. The promisee or the third party could assert control over the condition such that the promisor is obligated to perform, thus the promisor's promise is not illusory.

A gratuitous surety contract will be considered unenforceable for lack of consideration if the surety makes his promise to pay __________. Abefore the creditor has performed or made an absolute promise to perform Bafter the creditor has performed or made an absolute promise to perform Cas a response to a condition precedent to the creditor's performance Din exchange for payment from the creditor

B If a gratuitous surety (i.e., one who is not paid for his services) does not make his promise until after the creditor has performed or made an absolute promise to perform, there is no consideration to support the surety's promise because of the preexisting legal duty rule—the creditor has not incurred any new detriment in exchange for the surety's promise. Thus, the surety's promise is unenforceable. If the gratuitous surety makes his promise to pay before the creditor performs or promises to perform, the creditor's performance or promise will serve as consideration for the surety's promise. If the contract between the debtor and the creditor makes obtaining a surety a condition precedent to the creditor's performance, so that the creditor would be justified in refusing to perform the contract until a surety is obtained, the surety's promise is binding if the creditor performs in reliance on the surety's promise. As with other contracts, if the creditor gives additional consideration such as payment in exchange for the surety's promise, the surety will be bound.

An otherwise valid debt that is now barred by the statute of limitations can still be enforced if______. AThe debtor makes a new promise to pay the debt, either orally or in writing BThe debtor makes a new promise to pay the debt in writing CThe debtor admits owing the debt, either orally or in writing DThe debtor admits owing the debt in writing

B If an otherwise valid debt would be enforceable except for the fact that a technical defense to enforcement stands in the way (e.g., statute of limitations), the courts will enforce a new promise to pay if it is in writing or has been partially performed. An oral promise is not enough. The fact that debtor admits owing the debt, orally or in writing, is not sufficient, as that obligation is now barred.

Seller offers in writing to sell 1,000 widgets to Buyer for $10 per widget. Buyer replies in writing, "I accept. Please ship 1,000 widgets for $10 per widget, including shipping charges." Which of the following statements is true? AIf both Seller and Buyer are merchants, the shipping charges term is knocked out and replaced by a gap-filler BIf both Seller and Buyer are merchants, the contract includes the shipping charges term unless Seller objects within a reasonable time CIf Seller is a merchant and Buyer is a nonmerchant, the contract includes the shipping charges term unless it is considered a material alteration DIf both Seller and Buyer are nonmerchants, there is no contract

B If both the buyer and seller are merchants, an additional term in an acceptance is included in the contract unless it materially alters the original offer terms, the offer expressly limited the acceptance to its terms, or the offeror objects within a reasonable time. (The shipping charge does not materially alter the original offer.) If both Seller and Buyer are merchants, different (not additional) terms in an acceptance may be knocked out and replaced by a gap-filler. (The shipping charge is an additional term.) Under the U.C.C., a contract is formed even if terms of the acceptance do no match the terms of the offer. However, if either party is a nonmerchant, an additional term in an acceptance does not become part of the contract unless the offeror expressly agrees to it. Here, if both Buyer and Seller are nonmerchants, there is a contract, but the shipping charges term will not be included unless Seller expressly agrees to it.

When dealing with a contract for the sale of goods, in which of the following situations is a writing required for the contract to be enforceable? AThe party against whom enforcement is sought admits to the contract in court BThe contract price was originally less than $500, but was later modified to a price of $500 CThe goods were accepted by the buyer DThe goods are to be specially manufactured for the buyer and the seller has made substantial beginning in their manufacture

B In determining whether a contract is for $500 or more, Article 2 gives effect to any modification; thus even if the contract price was originally less than $500, if the contract is later modified to fall within the Statute, it must comply with the Statute's writing requirements to be enforceable. If goods are to be specially manufactured for the buyer and are not suitable for sale to others in the ordinary course of the seller's business, a writing is not required once the seller has made a substantial beginning in their manufacture or commitments for their purchase. If the party against whom enforcement is sought admits in pleadings, testimony, or otherwise in court that the contract was made, the contract is enforceable without a writing for the quantity of goods admitted. If goods are accepted, the contract is enforceable without a writing to the extent of the quantity of goods accepted.

Mutual mistake can be a defense to the formation of a contract if: AThe mistake concerns the value of the subject matter of the contract BThe mistake concerns a basic assumption on which the contract is made CThe adversely affected party bore the risk of the mistake DThe mistake has any effect on the agreed-upon exchange

B Mutual mistake can be a defense to the formation of a contract if the mistake concerns a basic assumption on which the contract is made. Such a mistake renders the contract voidable by the adversely affected party if the mistake has a material effect on the agreed-upon exchange (not any effect on the agreed-upon exchange) and the party seeking avoidance did not assume the risk of the mistake. Thus the answer choice stating that the adversely affected party bore the risk of the mistake is incorrect. A mistaken assumption as to the value of the subject matter generally will not be remedied.

Which of the following would not be considered valuable consideration that supports a contract? APeace of mind for the promisor. BFulfillment of a condition to receive a gift. CA benefit with no economic value. DThe gratification of influencing the mind of another.

B The mere fulfillment of a condition to receive a gift is not adequate consideration. The fulfillment of the condition must be of some benefit to the promisor to constitute proper consideration. The benefit to the promisor need not have economic value. Peace of mind or the gratification of influencing the mind of another may be sufficient to establish bargained-for consideration.

Under the Statute of Frauds, an oral contract for the sale of goods priced at $500 or more is unenforceable against the buyer. Which of the following scenarios is not an exception to this general rule? AThe goods were specially manufactured for the buyer BThe buyer admits to the oral contract in court pleadings CThe contract price was originally $450 but was later modified to $500 DThe buyer received and accepted the goods in question

C A $450 contract modified to $500 is not within any exception to the Statute of Frauds. The Statute of Frauds requires that certain contracts be evidenced by a writing signed by the parties sought to be bound. This includes contracts for the sale of goods for a price of $500 or more. In determining whether a contract is for $500 or more, Article 2 gives effect to any modification. Thus, even if the contract price was originally less than $500, if the contract is later modifies so as to fall within the Statute, it must comply with the Statute's writing requirements to be enforceable. If the party against whom enforcement is sought, in this case the buyer, admits in pleadings, testimony, or otherwise in court that the contract was made, the contract is enforceable without a writing (but in such a case the contract is not enforced beyond the quantity of goods admitted). If goods are either received and accepted or paid for, the contract is enforceable. However, the contract is not enforceable beyond the quantity of goods accepted or paid for. Thus, if only some of the goods called for in the oral contract are accepted or paid for, the contract is only partially enforceable. A contract for specially manufactured goods (i.e., goods that are to be specially manufactured for the buyer and are not suitable for sale to others by the seller in the ordinary course of his business) can be enforced without a writing when the seller has reasonably indicated that the goods are for the buyer and made a substantial beginning in their manufacture or committed for their purchase before notice of a repudiation was received. In this case, the question indicates that the manufacture of the goods was actually completed (by using the past tense "manufactured"); so the exception would apply.

Which of the following service contracts must satisfy the Statute of Frauds to be enforceable? AA contract for the client's personal care during an illness of unknown duration BA contract for a specific task that will take approximately 12 months to complete CA contract for one month of service that is to begin 13 months in the future DA contract for the lifetime of the client

C A contract that by its terms cannot be performed within one year is subject to the Statute of Frauds. The date runs from the date of the agreement and not from the date of performance. Thus, a contract for one month of service that is to begin 13 months in the future must satisfy the Statute to be enforceable. If the contract is possible to complete within one year, it is not within the one-year prong of the Statute of Frauds, even though actual performance may extend beyond the one-year period. A specific task that will take approximately 12 months to complete might be completed in less time. Likewise, a contract for the client's personal care during an illness of unknown duration might be completed in less than a year if the client recovers quickly. A contract for the lifetime of the client is not within the Statute because it is capable of performance within a year since the client could die at any time.

Under the doctrine of promissory estoppel, a promise is enforceable __________ when the promisor should reasonably expect to induce action or forbearance, and such action or forbearance is in fact induced. AProvided there is proper consideration BIn all cases CIf necessary to prevent injustice

C A promise is enforceable if necessary to prevent injustice when the promisor should reasonably expect to induce action or forbearance, and such action or forbearance is in fact induced. Such a promise is not enforceable in all cases because the doctrine of promissory estoppel only applies when necessary to prevent injustice. Promissory estoppel is considered a substitute for consideration. Thus proper consideration is not necessary if the facts indicate that the promisor should be estopped from not performing.

A promise not to sue on a claim can be considered valuable consideration only if: AThe claim is valid or would appear to be valid under the reasonable person standard BThe claim is valid, in law or in fact CThe claim is valid or the claimant reasonably and in good faith believes the claim is valid

C A promise not to sue on a claim can be considered valuable consideration only if the claim is valid or the claimant reasonably and in good faith believes the claim is valid. The reasonable person standard does not apply; the claimant herself must actually believe the claim is valid.

Which of these might be considered valuable consideration? AA promise to do something that one is legally obligated to do BA promise to make a gift CA promise with no economic value

C A promise with no economic value might be considered valuable consideration. Peace of mind or the gratification of influencing the mind of another may be sufficient, provided that the promisee is not already legally obligated to do the requested act. A promise to make a gift is not considered valuable consideration as no bargained-for exchange is present.

Which of the following statements about suretyships is false? ASuretyship contracts involve a promise to pay the debt of another BSureties can be compensated or gratuitous CSuretyship contracts are enforceable without consideration

C A suretyship contract is not enforceable unless it is supported by consideration. A suretyship contract involves a promise to pay the debt of another. A suretyship contract can be compensated or gratuitous. If a surety is compensated, the requirement of consideration is not much of an issue, because the compensation will serve as consideration for the surety's promise. If the surety is gratuitous, the timing of the promise becomes important in determining whether adequate consideration is present. If the surety makes his promise to pay before or at the same time as the creditor performs or promises to perform, there is consideration.

An agreement in which one party has become bound but the other has not, can be said to lack __________. Adetriment Bexclusivity Cmutuality Dsatisfaction

C An agreement in which one party has become bound but the other has not can be said to lack mutuality. Consideration must exist on both sides of the contract; i.e., the promises must be mutually obligatory.

What does it mean when an agreement lacks mutuality? AThe consideration for the agreement is not equal BThe agreement is so unfair to one party that it will be deemed unconscionable COne party has become bound but the other has not

C An agreement that lacks mutuality is one in which one party has become bound but the other has not. Consideration must exist on both sides of the contract. Without mutuality, there is consideration on only one side. Courts of law normally will not inquire into the adequacy of consideration to judge whether the agreement is unfair to one of the parties. If a party wishes to enter into a contract that others might judge unfair, so be it. (Note that courts of equity may inquire into the adequacy of consideration and deny an equitable remedy if the court deems the deal unconscionable.) While consideration must exist on both sides of the contract, there is no requirement that consideration be equal.

Which of the following statements is true regarding the modification of a contract? ABoth the common law rule and U.C.C. Article 2 allow modification without new consideration if the modification is sought in good faith BBoth the common law rule and U.C.C. Article 2 always require new consideration to modify a contract CThe common law rule generally requires new consideration to modify a contract, whereas U.C.C. Article 2 allows modification without new consideration if the modification is sought in good faith DU.C.C. Article 2 requires new consideration to modify a contract, whereas the common law rule allows modification without new consideration if the modification is sought in good faith

C At common law, modification of a contract generally is unenforceable unless it is supported by new consideration. U.C.C. Article 2 does not follow this rule. Under U.C.C. Article 2, contract modifications sought in good faith are binding without consideration

How can one avoid the preexisting legal duty rule? ABy full performance of the duty BBy making a brand-new identical promise CBy modifying the original consideration slightly DBy beginning performance

C Courts are anxious to avoid the preexisting duty rule, which states that the promise to perform, or the performance of, an existing legal duty is not consideration. Thus modifying the original consideration, even slightly, is generally enough to avoid the rule. Making a brand-new identical promise is not sufficient because there is no consideration for the new promise. There must be new consideration or the consideration that is different in some way, such as by accelerating performance, to avoid the preexisting duty rule. Beginning performance does not avoid the preexisting legal duty rule.Even full performance of a preexisting legal duty is not sufficient consideration. There must be some new or different obligation.

Under Article 2, which of the following is not a true statement concerning contract terms? AIf the price is not specified, the price is a reasonable price at the time of delivery. BIf the time for payment is not specified, payment is due at the time and place at which the buyer is to receive the goods. CIf a quantity is not specified, the quantity is a reasonable quantity under the circumstances at the time the contract was made. DIf the time for delivery is not specified, delivery is due within a reasonable time.

C If a quantity term is not specified, a reasonable quantity term will NOT be supplied. To form a valid sale of goods contract, there must be a quantity term. A quantity term will not be supplied by the court, and therefore, there is no gap-filling provision with respect to quantity. Under the Article 2 gap-filler provisions, if the price is not specified, the price is the reasonable price at the time of delivery. Similarly, if the time for delivery is not specified, the gap-filler provides that delivery is due within a reasonable time. The gap-filler provisions also provide that payment is due at the time and place at which the buyer is to receive the goods if the time for payment is not specified in the agreement.

If you agree to refrain from doing something that you have a legal right to do, you have __________. AIncurred a loss BProvided no consideration if you had no intention of doing the particular activity CSuffered a legal detriment

C If you agree to refrain from doing something that you have a legal right to do, you have suffered a legal detriment. A legal detriment need not involve any actual loss to the promisee or benefit to the promisor. A party suffers legal detriment if he agrees to refrain from doing something that he has a legal right to do—even if he has no intention of doing the act. For example, if a person of legal age agrees to refrain from drinking alcohol or smoking, he has suffered a legal detriment even if he was not a drinker or smoker. As long as he had a right to do the act he promised to refrain from, he suffers legal detriment.

What is the Statute of Frauds? AA statute that contains special rules for all contracts involving the sale of goods BA statute that allows a court to enforce a promise without consideration when necessary to prevent injustice CA statute requiring that certain types of agreements must be evidenced by a writing signed by the party sought to be bound

C In most instances, an oral contract is valid. However, under the Statute of Frauds, certain types of agreements must be evidenced by a writing signed by the party sought to be bound. Article 2 of the Uniform Commercial Code contains special rules for all contracts involving the sale of goods. Goods are defined as all thing movable at the time they are identified to be sold under the contract. Using the doctrine of promissory estoppel, a court can enforce a promise without consideration when necessary to prevent injustice when the facts indicate that the promisor should be estopped from not performing.

A writing is not required to enforce a contract that would otherwise be covered by the Statute of Frauds if: ABoth parties are merchants BBoth parties are nonmerchants CThe party against whom enforcement is sought admits to the existence of the contract in court DThe party seeking to enforce the contract offers evidence of prior dealings between the parties

C The Statute of Frauds requires that certain contracts be evidenced by a writing signed by the parties sought to be bound. However, if the party against whom enforcement is sought admits in pleadings, testimony, or otherwise in court that the contract was made, the contract is enforceable without a writing (but in such a case the contract is not enforced beyond the quantity of goods admitted). Prior dealings alone are not enough to remove a contract from the requirements of the Statute of Frauds. Just because the parties had an agreement in the past does not mean there is sufficient proof of a current agreement. In some such cases, estoppel could be applied where it would be inequitable to allow the Statute of Frauds to defeat a meritorious claim, but for that to occur the party seeking to enforce the agreement would need to show that enforcement is necessary to prevent injustice. An example of this would be if the other party falsely and intentionally told the plaintiff the contract is not within the Statute of Frauds or said he would reduce the agreement to a writing but failed to do so. The Statute of Frauds applies to both merchants and nonmerchants. There is no general exception to the Statute of Frauds just because both parties are merchants. Note, however, that under the merchant's confirmatory memo rule, in contracts between merchants, if one party, within a reasonable time after an oral agreement has been made, sends to the other party a written confirmation of the understanding that is sufficient under the Statute of Frauds to bind the sender, it will also bind the recipient if the recipient has reason to know of the confirmation's contents and does not object to it in writing within 10 days of receipt.

Which of these is the best example of a ratification? AA new valid promise to perform on a disputed debt BA new valid promise to perform a preexisting legal duty CA new valid promise to perform a voidable obligation

C The best example of ratification is a new valid promise to perform a voidable obligation, such as when an infant ratifies an already formed, yet voidable, contract upon reaching the age of majority, or when a defrauded person promises to go through with a tainted contract after learning of the fraud. A promise to perform a preexisting legal duty is not ratification because there is already a duty to perform an enforceable agreement. A promise to perform on a disputed debt will be enforced, as the compromise itself is considered consideration for the new promise. This is not an example of ratification because it does not involve a voidable obligation.

When a court refuses to enforce a provision of a contract to avoid "unfair" terms, it is applying the concept of: Apari delicto Bpromissory estoppel Cunconscionability Ddisaffirmance

C The concept of unconscionability allows a court to refuse to enforce a provision of a contract to avoid unfair terms. There are two types of unconscionability: substantive unconscionability based on lopsided terms, and procedural unconscionability based on unfair surprise or unequal bargaining power. A party might be able to successfully seek relief from a contract based on a defense of illegality, if the party is not in pari delicto, i.e., as culpable as the other party to the illegal contract. An infant might apply the concept of disaffirmance by choosing to disaffirm a contract he entered into before reaching the age of majority. Promissory estoppel allows a promise, unenforceable due to a lack of consideration, to be enforced if necessary to prevent injustice.

The implied warranty of merchantability is ______________. AImplied in every contract for the sale of goods by a merchant BImplied in every contract for the sale of goods CImplied in every contract for the sale of goods by a merchant who deals in goods of the kind sold

C The implied warranty of merchantability is implied in every contract for the sale of goods by a merchant who deals in goods of the kind sold. It is not implied in every sale of goods by any seller or even any merchant.

The Statute of Frauds requires: Athe handwritten signature of the party sought to be held liable on some document acknowledging the existence of the contract. Ba signed writing for suretyship promises that primarily serve the pecuniary interest of the promisor. Cone or more writings that reflect the material terms of the contract, signed by the person sought to be held liable. Da formal written contract signed by both parties to the agreement.

C To satisfy the Statute of Frauds, there must be one or more writings signed by the person sought to be held liable on the contract that reflect the material terms of the contract. The Statute of Frauds does not require a formal written contract signed by both of the parties. For example, a letter, receipt, or a check containing the material terms (e.g., quantity for sale of goods) and signed by the party to be charged satisfies the Statute of Frauds. The needed signature need not be handwritten, but the document or documents must include the material terms of the contract, not just acknowledge the existence of the contract. Generally, the Statute of Frauds requires that suretyship promises be in writing and signed by the party to be held liable. However, there is an exception for suretyship promises that primarily serve the pecuniary interest of the promisor; they are not within the Statute of Frauds.

Which of the following statements is true? AThe buyer may revoke acceptance of goods destroyed in a warehouse fire if they were defective, and the seller will bear the loss BIn a destination contract, the risk of loss is on the seller until the goods are tendered to the buyer at the destination, even if the goods are defective CIn a shipment contract, the risk of loss passes to the buyer when the goods are delivered to the carrier, unless the goods are defective

C Usually, the risk of loss passes to the buyer when the goods are delivered to the carrier, but if the goods are defective the risk remains on the seller. If goods are defective, the risk remains on the seller regardless of the type of shipping contract. Thus, the risk remains on the seller in a destination contract if the goods are defective. A buyer may not rightfully revoke acceptance once the goods are destroyed. A revocation is rightful only if it occurs before any substantial change in the condition of the goods which was not caused by their own defects.

A legal detriment can best be defined as __________. Anegotiating a bargained-for exchange Bmaking a promise to perform, or the actual performance of, an existing legal duty Csuffering an actual loss in exchange for some perceived benefit Ddoing something you are under no legal obligation to do, or refraining from doing something that you have a legal right to do

D A legal detriment will result if the promisee does something he is under no legal obligation to do or refrains from doing something that he has a legal right to do. Legal detriment need not involve any actual loss to the promisee or benefit to the promisor. For example, a party suffers legal detriment if he agrees to refrain from doing something that he had no intention of ever doing. As long as he had a right to do the act he promised to refrain from, he suffers legal detriment. A legal detriment is not a negotiation. It is also not the same thing as a bargained-for exchange, although it may be an element of such an exchange. A legal detriment is not a promise to perform, or the actual performance of, an existing legal duty. Legal detriment does not require performance; it can consist of refraining from doing something. Also, if a party has an existing legal duty to do something, he does not suffer a detriment by agreeing to do it.

An implied warranty of merchantability may be disclaimed __________. Aonly if the buyer has an opportunity to inspect the goods before entering into the contract Bonly by a written, conspicuous disclaimer that specifically mentions merchantability Cat any time before the goods are used Deither by a specific disclaimer mentioning merchantability or by general language such as "as is"

D An implied warranty of merchantability may be disclaimed either by a specific disclaimer mentioning merchantability or by general language, such as "as is." Under Article 2, the warranty of merchantability can be specifically disclaimed or modified only by mentioning merchantability. However, unless the circumstances indicate otherwise, an implied warranty of merchantability can be disclaimed by expressions, such as "as is," "with all faults," or other expressions that call the buyer's attention to the fact that there are no implied warranties. To be effective, the disclaimer need not be in writing. A written, conspicuous disclaimer is necessary only if the sales contract is in writing. Also, a specific disclaimer is not the only way to disclaim the implied warranty of merchantability. General disclaimer language is also effective. An implied warranty of merchantability cannot be disclaimed at any time before the goods are used. To be effective, a warranty disclaimer must be agreed to during the bargaining process. A thorough inspection or refusal to inspect by the buyer can create a general disclaimer of warranties, but an opportunity to inspect the goods is not a precondition to disclaiming the implied warranty of merchantability

In most states, when forming a contract, which of the following is not a necessary element of consideration? AA bargain for something of legal value. BA bargained-for exchange between the parties. CA benefit to the promisor or a detriment to the promisee. DAn economic benefit to the promisor.

D Basically, two elements are necessary to constitute consideration: (i) there must be a bargained-for exchange between the parties; and (ii) that which is bargained for must be considered of legal value or, as it is traditionally stated, it must constitute a benefit to the promisor or a detriment to the promisee. The benefit to the promisor need not have economic value; for example, peace of mind may be sufficient for consideration

Which one of the following contracts is considered void, rather than voidable? AA contract made by a mentally incompetent person BA contract made by an infant CA contract induced by duress or coercion DA contract involving illegal consideration

D Illegal consideration or subject matter renders a contract void and unenforceable. Contracts may be illegal because they are inconsistent with the Constitution, violate a statute, or violate public policy. Contracts induced by duress or coercion are voidable and may be rescinded as long as not affirmed. A contract entered into by a mentally incompetent person (i.e., one whose mental capacity is so deficient that he is incapable of understanding the nature and significance of a contract) is voidable. Mentally incompetent persons may affirm or disaffirm the contract during a lucid interval. Infants lack the capacity to enter into a contract binding on themselves. A contract entered into by an infant is voidable by the infant.

Which of the following normally would not be an exception to the preexisting legal duty rule? AA compromise based on an honest dispute as to duty. BA minor's ratification of a contract upon reaching the age of majority. CAn acceleration of the performance of the duty. DPayment of a smaller sum to settle an existing debt.

D In the case of an existing debt, payment by the debtor of a smaller sum than due will not be sufficient consideration for a promise by the creditor to discharge the debt. However, because courts are anxious to avoid the preexisting duty rule, payment of a smaller debt may be sufficient consideration if the payment is in any way different (e.g., stock instead of cash) or if the debt was honestly disputed. Almost any variation, such as accelerating performance, is considered adequate consideration. A promise to perform a voidable obligation (e.g., a minor's ratification of a contract upon reaching the age of majority) is also enforceable despite the absence of new consideration. If the scope of the legal duty owed is the subject of honest dispute, then a modifying agreement relating to it will ordinarily be given effect

Which of the following is not traditionally an element of the doctrine of promissory estoppel? ADetrimental reliance BThe reasonable expectation that the promise will induce action or forbearance CAn action or forbearance that is in fact induced by a promise DValuable consideration on both sides of the bargain

D Promissory estoppel is considered a substitute for consideration. Thus, consideration is not necessary if the facts indicate that the promisor should be estopped from not performing. A promise is enforceable if necessary to prevent injustice if: (i) the promisor should reasonably expect to induce action or forbearance; and (ii) such action or forbearance is in fact induced. Detrimental reliance is simply another way of describing the action or forbearance of the promisee (i.e., he relied on the promise to his detriment)

__________ can serve as a substitute for consideration. AA suretyship promise BA bargained-for exchange CAn option DPromissory estoppel

D Promissory estoppel is considered a substitute for consideration. Under the doctrine of promissory estoppel, a promise is enforceable even without consideration if necessary to prevent injustice if the promisor should reasonably expect to induce action or forbearance, and such action or forbearance is in fact induced. A bargained-for exchange is a necessary element of consideration, not a substitute for consideration. A suretyship promise is a promise to pay the debt of another. Suretyship contracts require proper consideration to be enforceable. An option is a distinct contract in which the offeree gives consideration for a promise by the offeror not to revoke an offer. An option is not a substitute for consideration.

The confirmatory memo rule applies __________. ATo all sales contracts covered by Article 2, whether or not the parties are merchants BAs long as the sender of the confirmatory memo is a merchant CAs long as the recipient of the confirmatory memo is a merchant DOnly if both parties to the agreement are merchants

D The Statute of Frauds requires that contracts for the sale of goods for a price of $500 or more be evidenced by a writing signed by the parties sought to be bound. The confirmatory memo rule is an exception, which allows a contract to be enforceable without a writing signed by the party to be bound, in this case the recipient of the confirmatory memo. Under the confirmatory memo rule, in contracts between merchants, if one party, within a reasonable time after an oral agreement has been made, sends to the other party a written confirmation of the understanding that is sufficient under the Statute of Frauds to bind the sender, it will also bind the recipient if he has reason to know of the confirmation's contents and does not object to it in writing within 10 days of receipt.

When two merchants enter into an oral contract for the sale of goods and one party sends to the other party a signed, written confirmation of the agreement, it: ABinds only the sender because it is signed only by the sender BBinds both the sender and the recipient, provided the recipient actually read the document and did not object in writing within 10 days of receipt CBinds both sender and the recipient, provided the recipient signs the confirmation within 10 days of receipt DBinds both the sender and recipient, provided the recipient had reason to know of its contents and did not object in writing within 10 days of receipt

D Under the confirmatory memo rule, in contracts between merchants, if one party, within a reasonable time after an oral agreement has been made, sends to the other party a written confirmation of the understanding that is sufficient under law to bind the sender, it will also bind the recipient if he has reason to know of the confirmation's contents and does not object to it in writing within 10 days of receipt. The recipient need not actually read the document; it is binding if the recipient had reason to know of its contents. Even though the memo is generally signed only by the sender, the recipient is bound; the recipient's signature is not necessary.

When the amount due on a debt is undisputed, which of the following will not be considered sufficient consideration for a promise by the creditor to discharge the debt? APayment before maturity. BPayment in a different medium. CPayment to one other than the creditor. DPayment of a smaller sum than due.

D When the amount due is undisputed, payment of a smaller sum than due will not be sufficient consideration for a promise by the creditor to discharge the debt. Neither a legal detriment nor a benefit would be present. In contrast, if the consideration is in any way new or different, such as payment before maturity or to one other than the creditor; or payment in a different medium (e.g., stock instead of cash), then sufficient consideration may be found.


Set pelajaran terkait

Chapter 13: Money, Banks, and the Federal Reserve

View Set

JavaScript outcome interview questions

View Set